Identity (Identity)

by dhw, Saturday, August 15, 2009, 11:27 (5361 days ago)

There are different discussions on different threads at the moment concerning free will, consciousness, morality, and other aspects of what it is that makes us not only human, but also different from one another. I'd therefore like to link them to "identity", in the hope that someone might come up with answers to some puzzling questions. - Matt has pointed out that thoughts often come into our heads seemingly of their own accord, with dreams as an extreme example. He suggests that since these thoughts come involuntarily, they must throw a degree of doubt on our free will. David argues that since our conscious and our unconscious minds are part of us, we can hardly attribute them to anything outside ourselves, and he believes that he is in control of who he is. BBella has put forward a concept of a possible Universal Intelligence which occupies all of us and therefore presumably thinks our thoughts for us, although we are not aware of it. (I hope I've got all this right, but no doubt you will correct me if I haven't.) I'm sure we all believe we have our own identity, so just what is it that makes us "us"? - Materialists would presumably say that the basis of identity is the physical stuff that goes to make up our bodies. I don't think anyone would question that the body has an enormous influence on our identity, and it's clear from the effects of drugs, accidents, operations, illnesses that a change in the body can bring about a change in the identity. But ... I'd like to underline this ... no-one has yet explained how the materials actually produce the thoughts that make us what we are. So are we servants of those materials? If not, what gives us control over our thoughts when we do control them, and what produces the thoughts we don't control? - Matt has talked of a tabula rasa. But is there such a thing? If the mind is controlled by the materials we're born with, the answer has to be no (because the core of the identity is already there). If it's not, why do some experiences leave their mark and others don't? Young children may not be sufficiently developed to make conscious choices or to produce calculated reactions, but they do choose and they do react in their own individual way. And so again, the basic identity is already there, and there is no tabula rasa. But if that core of identity is not material, what is it? (Just to clarify here, I'm not pleading for a "supernatural" explanation. I'm just asking questions.) - To sum it up, I understand that heredity, environment and chance all combine to make me who I am. What I do not understand is the mechanism that moulds these different influences into "me". If the mechanism is the material self with which I was born (heredity), all my responses to environment and chance have been pre-programmed. But if I have a degree of control over who I am and what I do ... and that, I'm sure, is what most of us believe ... what is the source of the conscious will that gives me that control?

Identity

by George Jelliss ⌂ @, Crewe, Saturday, August 15, 2009, 19:57 (5361 days ago) @ dhw

Some immediate responses to points in dhw's post: - "free will". This term tends to be used carelessly. I take the view that by thinking about a subject and coming to conclusions we can as a result influence our subsequent behaviour. Is this "free will"? In other respects our scope for wilful action seems to me to be somewhat limited by our circumstances. - "Matt has pointed out that thoughts often come into our heads seemingly of their own accord, with dreams as an extreme example." In my experience dreams are usually about ideas or events that have come up during the day, and they represent a sorting out process by which the brain tries to file them away and make sense of them. - "David argues that since our conscious and our unconscious minds are part of us, we can hardly attribute them to anything outside ourselves," Agreed. - "and he believes that he is in control of who he is." There I'm not so sure. I don't think I'm the same person I was ten or twenty or forty years ago, or even possibly a minute ago, since new thoughts have occurred to me. What "I" am evolves. - "I'm sure we all believe we have our own identity, so just what is it that makes us "us"?" I would just put it down to continuity. There is a series of connections between the person I am now and who I was forty years ago. - "Materialists would presumably say that the basis of identity is the physical stuff that goes to make up our bodies." I'm afraid that idea is sunsustainable, since the material of which we are composed, i.e. the atoms, is changing all the time. I did read once how long it took for all the atoms in our bodies to recycle, and it wasn't very long, but I forget the details. - "But ... I'd like to underline this ... no-one has yet explained how the materials actually produce the thoughts that make us what we are." I'm not an expert on neurology, but the way I understand it, thoughts occur in our brains in the form of connections between the neurons, or possibly as electric or chemical potentials. What these connections "mean" of course depends on the experiences that formed them, which they serve to remind us of by replaying the experience in some form. - "So are we servants of those materials?" No, not of the materials, but of the experiences and connections that formed us. - "If not, what gives us control over our thoughts when we do control them, and what produces the thoughts we don't control?" There are parts of our brain where we have some control over our thoughts (probably the frontal lobes) and there are other parts that work subconsciously, as in dreams. - "Matt has talked of a tabula rasa. But is there such a thing?" That's an outdated psychological notion, due to Locke I think. The modern view (see Pinker) is that babies are born with innate capacity for their brain to develop rapidly given a helpful and stimulating environment such as is provided by loving parents. Bad experiences can lead to maldevelopment. - "Young children ... do react in their own individual way." Only after a period of development I would think. - "What I do not understand is the mechanism that moulds these different influences into "me"." Mechanism is one of your fetish words! No machine is needed, just individual experiences. Why would you develop into someone else!

--
GPJ

Identity

by David Turell @, Saturday, August 15, 2009, 23:56 (5361 days ago) @ George Jelliss

"free will". This term tends to be used carelessly. I take the view that by thinking about a subject and coming to conclusions we can as a result influence our subsequent behaviour. Is this "free will"? In other respects our scope for wilful action seems to me to be somewhat limited by our circumstances. - You are right. Our 'free will' is limited by our societal relationships, but we are free to make correct choices in our dealings with others. - > "and he believes that he is in control of who he is." There I'm not so sure. I don't think I'm the same person I was ten or twenty or forty years ago, or even possibly a minute ago, since new thoughts have occurred to me. What "I" am evolves. - > "I'm sure we all believe we have our own identity, so just what is it that makes us "us"?" I would just put it down to continuity. There is a series of connections between the person I am now and who I was forty years ago. - I don't understand why you state "I'm not so sure" and then describe the continuity. I agree that our personality is put together like a layer cake, but there are continuing guidelines and it is a contiguous whole, hopefully with a deepening layer of 'maturity' and understanding what life is about. - 
 
> "Materialists would presumably say that the basis of identity is the physical stuff that goes to make up our bodies." I'm afraid that idea is unsustainable, since the material of which we are composed, i.e. the atoms, is changing all the time. I did read once how long it took for all the atoms in our bodies to recycle, and it wasn't very long, but I forget the details. 
 
But our recycling in under the control of the DNA code. Each cell has it. We just don't change that much. Recycling rates are different for different tissues. Bones and teeth hardly at all. Nails, hair and skin rapidly. In younger folks the plasticity of the brain is enormous. It grows from 1/4th of body size and is 1/8th of adult size. Molding of the connections continues until about age 24,and then memories are laid down until death, as are ideas and conclusions. The means for memory is still barely understood. - 
> "If not, what gives us control over our thoughts when we do control them, and what produces the thoughts we don't control?" There are parts of our brain where we have some control over our thoughts (probably the frontal lobes) and there are other parts that work subconsciously, as in dreams. - I agree. 
 - 
> "Young children ... do react in their own individual way." Only after a period of development I would think. - Not exactly a long period of development. My two daughters were entirely different people by age two, and all of that is still seen today, like a 90 degree fork in the road.

Identity

by dhw, Monday, August 17, 2009, 12:46 (5359 days ago) @ George Jelliss

George and David have replied to my post, but interestingly it's David's post on evolution (15 August at 14.20) that comes closest to approaching (though not solving) the problem I'm grappling with: "One of the biggest jumps in evolution was the development of the nerve cell, the ability of living animal material to produce ions that would travel along branches of the cell and impart information that created an activity." And the ant brain "is of itself a tiny living computer made up of nerve cells." I'll come back to this in a moment. - George, I agree with most of your comments, but they end one level away from what I'm questioning. You say dreams "represent a sorting out process by which the brain tries to file them away and make sense of them." But are the dreams and the brain independent of you? Don't answer that yet. You say you're not sure if you're the same person as you were, possibly a minute ago, "since new thoughts have occurred to me". Where did those thoughts come from? Don't answer that either. You go on: "thoughts occur in our brains in the form of connections between the neurons, or possibly as electric or chemical potentials." Do the neurons, or electric or chemical potentials activate or control themselves? Don't answer that either, because now we come to the crunch. - You say: "There are parts of our brain where we have some control over our thoughts (probably the frontal lobes) and there are other parts that work subconsciously, as in dreams." David agrees. So do I. And that is where my problem begins. What is this "we"? WHAT controls the thoughts? What is it that intentionally impels the ions to "travel along branches of the cell and impart information that create an activity"? What makes the "tiny living computer" function? And as regards the parts that work subconsciously, what is it that involuntarily sparks the electricity and translates it into thoughts and images? - I tried to describe the problem when I wrote: "What I do not understand is the mechanism that moulds these different influences into 'me'". You responded: "Mechanism is one of your fetish words! No machine is needed, just individual experiences. Why would you develop into some else!" And yet, George, you don't think you are the same person you were possibly even a minute ago. The experiences are the influences, and I'm asking what it is that processes them in the individual manner that is ours. I agree when you say there is continuity, and I agree when you say that what "I" am evolves. My question concerns the source of that part which is continuous. That "we" have some control, "we" process our experiences, "we" think thoughts, or thoughts occur to "us" is a statement of fact, but explains nothing. The mystery is not what tools are used, but how they work and, above all, what is using them.

Identity

by BBella @, Monday, August 17, 2009, 18:48 (5359 days ago) @ dhw
edited by unknown, Monday, August 17, 2009, 19:31

You [George] say: "There are parts of our brain where we have some control over our thoughts (probably the frontal lobes) and there are other parts that work subconsciously, as in dreams." David agrees. So do I. And that is where my problem begins. What is this "we"? WHAT controls the thoughts? What is it that intentionally impels the ions to "travel along branches of the cell and impart information that create an activity"? What makes the "tiny living computer" function? And as regards the parts that work subconsciously, what is it that involuntarily sparks the electricity and translates it into thoughts and images? - Of course everything that sparks is sparked by the sun, or better said the play of the light with the dark. Everything comes from the light/dark source, because without darkness, there is no light (said the darkness). So, everything comes from light playing in/with the dark. - My brother once pondered, which came first, the dark or the light? He concluded, one could only exist because of the other. So, possibly, there once existed dark and light, and thru time, they both happily agreed to mix and mingle and play well together creating what WE are. But one day, creation asked; where did the spark of idenity come from first which said, "I am?" The light of course, being mouthy said, twas me, I am creator of all that IS. The dark, of course, said...uh...excuse me? - Could it have been the light which spoke first? And really, why would it matter who spoke first because one is not without the other (the dark says and the light must agree!) Of course, once the question was spoken, who came first, or whose idea was it first to speak, the dark refused the lower state and said I am One, for me you would have no one to have spoken to!!! Only makes sense!!! - So, here we be...the long held fight of the dark and the light...I guess the light would win if the light could do without the dark because the light spoke first (if it did). But, as the darkness said, there would have been no one to speak to if there were no darkness. So, we must conclude, both are interdependent, if they choose to continue along this journey of WE (creation). But alas...they have no choice, as they are, and cannot not be. - >The mystery is not what tools are used, but how they work and, above all, what is using them. - The tools are: dark and light, which we know. How do they work? they intermingle creating light and shadow...which makes up all that IS. What is using these shadows? We are. Who is/are we? Dark and light. The circle of life. Seems simple enuff-right?). What first sparked the dark and the light to create? Who knows...maybe they were always there creating together, but the fighting began for the number one spot when light and dark created man (which created religion). But, hopefully, as they are evolving, they are coming to realize there never was a #1....for without the one the other could not be. Fitting for an ending isn't it? And so, they lived happily ever after...or did they? I can just hear the light saying...I spoke to you first...that makes me first! nanananana.... Mommy wisdom says... Give it up light!!! Ok...so you are the brightest tool in the shed...i will give you that!!! But without brother darkness there is no shed!!! K???? Now, go out and play... - Ok...suddenly, all the religions make perfect sense....

Identity

by BBella @, Tuesday, August 18, 2009, 20:36 (5358 days ago) @ BBella

[dhw]What is this "we"? WHAT controls the thoughts? What is it that intentionally impels the ions to "travel along branches of the cell and impart information that create an activity"? What makes the "tiny living computer" function? And as regards the parts that work subconsciously, what is it that involuntarily sparks the electricity and translates it into thoughts and images?
> - Thinking more about the question: - The play between light and dark possibly creates the illusion of stuff, or the material, thru the play of shadow. The colors we see thru the prism of the eye as well as the higher vibrations of the physical create the illusion of our world and life. Thought and language is just as much a part of this illusion and probably didn't come to be until it was necessary for communication when beings who wanted to communicate thru voice began language. Thinking about it more, I do not see light and dark speaking or having language or even the need for language communication, until man was created. Then of course, once man began communicating, they also began to question. So here we be, asking questions. If there are aliens or angels, other beings, etc., it is always said they do not "need" voice or language for communication. There is very likely a much older way of communicating that we as humans have (as we are all a part of that which makes up everything) but no longer primarily use or recognize. - As for thoughts traveling in and out of our mind field; thoughts travel on wavelengths just as radio waves and other communications travel. I have no doubt in my mind that when stuff pops in my head from nowhere, sometimes it's normally from these traveling voice and picture transmissions thru the airwaves. Of course there are thoughts in the mind that bubble up from the subconscious as well, but I know for sure language and pictures travel on airwaves. I many times know when someone is about to call me, or when something happens to a celebrity/known personality, like a politician, and even sometimes tragic events before I even read or hear about them, because the picture or person will float into my mind before I even hear about it. Of course I rarely know any details, only that something is going on with this or that person or place. This also happens on a regular basis for one person in my family even more than for me. Her problem too is that she is constantly awakened by hearing a radio that isn't there. - I think when our minds are at rest we are more open to these radio waves...and on a subliminal level, to certain things that are going to happen, as that information is traveling our way on vibrational radio waves. What makes us the target for these wavelengths is another question. Maybe it's simply that we are connected or open in some way thru knowledge about this info. I know there are a few times someones name will come to mind, or a picture of something, or an event that I don't know, but this is extremely rare. Most of the time it is a connection to things or people I either do know or know about. As is said, it takes one to know one. If you know nothing about something it is more unlikely that your mind is an open receiver to receive what you don't already know something about. I am definitely a very open minded person so maybe I receive stuff I am not familiar with because of this. Of course it does no good whatsoever to know something about someone you don't know, or about something you can do nothing about. One reason I would just rather not know. - Just more ramblings.

Identity

by George Jelliss ⌂ @, Crewe, Wednesday, August 19, 2009, 13:04 (5357 days ago) @ BBella

BBella, a lot of what you write, such as about light and dark, can be taken to be a form of poetry, but the trouble is you don't seem to have any lines of demarkation to distinguish between that mode of thought and reality. It's when you start making factual claims such as "thoughts travel on wavelengths just as radio waves and other communications travel" which have no basis in science that I have problems. The experiences you describe are subjective. Your claims to have preknowledge of future events are too vague for any kind of scientific analysis.

--
GPJ

Identity

by George Jelliss ⌂ @, Crewe, Monday, August 17, 2009, 21:46 (5359 days ago) @ dhw

dhw's last post was rather convoluted, but I'll try to answer at least in part, though I'm trying to clarify my thoughts as I go along. - dhw cites DT describing an ant brain as "a tiny living computer made up of nerve cells.". I'm not keen on comparing brains or nervous systems to computers too closely, though it sometimes helps. - dhw asks: "are the dreams and the brain independent of you?" ... "Where did those [new] thoughts come from?" ... "Do the neurons, or electric or chemical potentials activate or control themselves?" ... "What is this "we"? WHAT controls the thoughts? What is it that intentionally impels the ions to "travel along branches of the cell and impart information that create an activity"? What makes the "tiny living computer" function?" - I would say that most of my thinking is not of the "intentionally impelling" sort, in which "I", the ego, the will, acting through the frontal lobes (or something like that) directs the course of events. That would be just too exhausting! The "I" in this sense is only part of the "Me", if that makes sense. - dhw: "And as regards the parts that work subconsciously, what is it that involuntarily sparks the electricity and translates it into thoughts and images?" - The human brain works by taking in input through the senses, and from memories of past experiences and shuffling them around, comparing them and sorting them out in ways that hopefully make sense (pattern finding). I imagine a lot of that is done out of habit or are processes (macros?) that have evolved and proved useful. A lot of those can work subconsciously. - dhw: "you don't think you are the same person you were possibly even a minute ago. ... I'm asking what it is that processes them [experiences and thoughts] in the individual manner that is ours." - We are what we are, as individuals because of our inherited DNA and our life-experiences. You could say "we" are the sum of our experiences, of all our history. - dhw: "I agree when you say there is continuity, and I agree when you say that what "I" am evolves." - I'm not sure that continuity is the right word. Perhaps I should have said "contiguity". Continuity makes it seem too smooth. I've tended to make large sudden discontinuous changes to my life, such as my move back to Hastings last December. - dhw: "My question concerns the source of that part which is continuous. That "we" have some control, "we" process our experiences, "we" think thoughts, or thoughts occur to "us" is a statement of fact, but explains nothing." - On the contrary I find it a sufficient explanation. We are our experiences. I remember reading some science fiction stories in which it was suppose to be possible for people to make copies of themselves, so that if they got killed their backup copy would be activated and take their place. the backup would have all the same memories up to a certain time before. If you met such a clone of yourself, which one would be the "real" you? The answer would be that they were originally both you, but as soon as they start having their own separate experiences, they diverge to become two separate beings.

--
GPJ

Identity

by dhw, Wednesday, August 19, 2009, 07:52 (5357 days ago) @ George Jelliss

I am trying to pinpoint "identity", and both George and BBella have gallantly attempted to come to my rescue. - The symbiotic relationship BBella describes between dark and light is a great image for vast areas of life, but it's your earlier post that asks questions relevant to what I'm trying to figure out: "how did ONE come to have thought in the first place...?" "What sparked ONE to make the choice...?" Only I'm focusing on us rather than on ONE. George is also touching on the essence of the problem when he writes: '"I", the ego, the will, acting through the frontal lobes [...] directs the course of events. The "I" in this sense is only part of the "Me", if that makes sense." - Everything you write, George, makes perfect sense, and I am the one who is struggling to convey something that language actually conceals. We simply take it for granted that we are an "I", we have a "will", an "ego", just as we take it for granted that we have consciousness, subconsciousness, emotions, memory, imagination etc. But these words denote something so complex that we can scarcely begin to understand how they function. Cells, chemicals, neurons, ions, electrical impulses may be part of a physiological process that goes on when we exercise these faculties, but they do not explain how matter becomes conscious of itself, feels emotions, remembers, imagines. Nor do they explain what is the "I" that can stop or start the physiological processes. - You say we are our experiences. But we are also our bodies. And we are our minds. If pressed to say which of these was the seat of our identity, I would say it is the mind. This will have been greatly influenced, and is continually being influenced, by the body and by experiences, but it is the mind that decides what attitude or what course of action to take. Is the mind the same as the brain? Is it nothing but a collection of cells, i.e. also part of the body? If so, we are the servants of our cells. But we don't think we are. We think we have an I/ego/will that enables us to exercise a degree of control over ourselves, to give orders to ourselves, to watch and criticize ourselves. But if the I/ego/will is not physical cells, what is it? - I obviously can't answer my own question, but no doubt a religious person would say that it's the "soul". That too is a word which explains nothing, but rather like "dark matter" and "dark energy" at least it gives us a term for an area of existence we do not understand. BBella suggests that if there is a Universal Intelligence, it might not understand either. Ah well, that would put me in good company!

Identity

by George Jelliss ⌂ @, Crewe, Wednesday, August 19, 2009, 13:12 (5357 days ago) @ dhw

dhw asks: "Is the mind the same as the brain? Is it nothing but a collection of cells, i.e. also part of the body?" - I'm surprised you can ask such a question after all the discussion. I'm happy to identify the mind with the activity of the brain.

--
GPJ

Identity

by dhw, Friday, August 21, 2009, 13:12 (5355 days ago) @ George Jelliss

George is "happy to identify the mind with the activity of the brain." - BBella believes that thoughts travel through airwaves. George points out that such claims have no basis in science. - I have no difficulty understanding that without the brain, I will have no consciousness, emotion, imagination, reason, thoughts, memory etc. Without a TV set, I won't receive sound or picture. But the TV set isn't the source of the sounds or pictures, any more than the primary auditory cortex and the primary visual cortex are. Sensory impressions come from outside and are processed within the posterior cortex, and these processes are automatic. The frontal lobe is the seat of all our planning and our abstract thinking, but where do the thoughts and control of thoughts come from? These are not automatic processes. So what intentionally switches the processes on and off, and what controls them? The answer, of course, is "I" do, but this is the focus of our discussion. Is the "I" nothing more than my cerebral cortex? I looked up "cerebral cortex" on Wikipedia: it's "a structure within the brain that plays a key role in memory, attention, perceptual awareness, thought, language, and consciousness." Playing a key role does not constitute the whole show. I'm trying to find out what is involved in actually running the show. - BBella's concept also makes us into receivers not only of sensory impressions but also of thoughts. Even if George is right, and all these processes are purely physical, I don't see why at least theoretically the impulses of thought shouldn't join all the other waves rushing through the air, but as with theories like abiogenesis, I'm inclined to wait till science comes up with some proof. However, if one did accept the idea of thought waves, they still wouldn't explain the origin of thoughts. A Universal Intelligence thinking for all of us is no less mysterious than brain cells creating as well as controlling our thoughts, especially if the UI doesn't understand the process either! - As for your own mysterious connections, the fact that "the experiences you describe are subjective" doesn't disturb me as much as it does George, since I'm far more open to accepting personal experience as evidence. In any case, it seems to me that ultimately we CAN only base beliefs on subjective factors. Belief that science will one day come up with all the answers is also subjective. Whether we can convince others is another matter, but that's why we're having these discussions.

Identity

by BBella @, Monday, August 17, 2009, 18:07 (5359 days ago) @ dhw
edited by unknown, Monday, August 17, 2009, 18:17

[bbella wrote from Arguments against Design] None of us hold or have any identity apart from that which we relate with and to. In other words, we are nothing without each other.
>>[dhw responded]I think this is only partly true. We have relations with ourselves as well as with others, and these depend largely on our own characteristics. Perhaps, though, you can develop this idea on the new "Identity" thread. - If I existed within a blank space with nothing but myself to relate to, no matter how long, I would very unlikely ever exhibit anything but nothingness. I would have no reason to move, talk, feel, etc. Without any surroundings I am nothing. Without something (any-thing!) to relate to, I would not know what I am or what I am capable of. Perhaps it's not by chance you chose "Identity" as title for this thread? For without what we now consider the uni-verse, I/God/UI/We, our surroundings, we would be nothing and have no identity...maybe that is what UI was shooting for when it first replicated? It is as if we see this TRUTH with our microscope on a grand scale within life itself happening on an ever continual basis. Even within the stories of old, we get this thought...there is one, and one decides it is lonely and so from one creates two and so here we are. It some way this is totally mythical, yet, we see it happening every day in life. Of course, this is also the crux....how did ONE come to have thought in the first place and how did ONE realize it had the ability to become TWO...this is the real question here, right? And, low and behold...maybe UI actually does not have an answer for this question and is searching to understand itself? Who knows? - >maybe it doesn't matter whether it (free will) is or isn't free, but the question I would like to pursue is the source of the ability to make the choice, because it's that source (whatever it may be) that shapes our identity. - Again, this is the crux of the question....even if the source, we could call it the ONE, was, then what sparked ONE to make the choice to become two and so on? - Good question. - The only thing that comes to mind is, because it realized it could?

Identity

by xeno6696 @, Sonoran Desert, Monday, August 24, 2009, 06:09 (5352 days ago) @ dhw

dhw, - I have many thoughts about this topic, I'll do my best to keep everything straight. I have two central observations: - 1. I recognize I have the power to do what I want to do. - 2. I recognize that what I want to do, often comes upon me unbidden. - It is the interplay between these two that gives me the difficulty I have in the ages-old debate of free will. - I view consciousness as simply the ability to make decisions, in the case of humans this is aided by the use of symbolic language. - I probably oversimplify David's position, but to me an absolute assertion of free will cannot be taken at face value. I agree with him that the thoughts do not come from outside, therefore they must be mine, but the *will* part of free will is what compels me to say "not completely." We have strong internal drives, and these drives are what move us. What drives us shapes our personality, but its the simple choices we make that alter the drives, makes them weaker or stronger. - I often like to say "Man is a torrent. One must master the currents for any hope of mastering the self." When I was on vacation as mentioned in my return post, I felt compelled to read Emerson. I brought a couple of other books, yet I returned to Emerson to read and reread. In analyzing this, yes I had free will to read something else, and I know I could have. However, I knew that doing so would have been fruitless. My mind would not have been up to the task. I find that for me, "free will" has been an exercise in nudging my drives where I want them to go. Just now I had to pause for five minutes to join my wife in watching the Tudors for the scene were Anne Boleyn lost her head. I could have chosen to continue watching, yet I couldn't--drawn in by the melodrama of excellent tele writing. I wasn't even aware that I wasn't typing. - I regret that this doesn't answer too many questions, but perhaps should let people see what I mean when I can't fully say that I'm in control. I am *most* of the time. I think that the doctrines that says that we control ourselves at all times understimates human instinct/drive and overestimates abstract reasoning--another relic of Thomist thought. It needs to go. - The best things mankind have made weren't deliberate. They started as crude and unbidden thoughts that were whittled by deliberate will. Claiming absolute free will negates inspiration just as much as claiming we are slaves to atoms negates free will. Neither tells a complete truth.

--
\"Why is it, Master, that ascetics fight with ascetics?\"

\"It is, brahmin, because of attachment to views, adherence to views, fixation on views, addiction to views, obsession with views, holding firmly to views that ascetics fight with ascetics.\"

Identity

by dhw, Tuesday, August 25, 2009, 07:55 (5351 days ago) @ xeno6696

First of all, Matt, welcome back to AgnosticWeb. It's good to hear that you and your wife are recharged and rejuvenated. With your positive attitude, you will overcome all the remaining problems. - While you were away, David has been providing us with fascinating glimpses of the extraordinary ingenuity of "Nature's I.Q.". I'd like to link these with the subjects of identity as well as evolution (certain aspects of which David is challenging). - First, though, another word about animals. There was a sad photograph in this week's Sunday Times, and the caption reads: "Two-year-old orphan Jumaane nudges the face of his dead mother in an attempt to bring her back to life. Yoki, who at 19 was not elderly for an elephant, died on Thursday at the Nyiregyhaza animal park in Hungary, apparently from a tumour in her spleen. Keepers say that since his mother's death, Jumaane has been inconsolable, wandering around aimlessly and weeping." - There is nothing unusual in this. Elephants not only grieve, but they even stand and contemplate places of death when they return years later. Conversely, they can also celebrate wildly, trumpeting, dancing, and peeing with excitement. If we have common descendants, we should not be surprised that we have common characteristics (which may include the peeing!), and these are not confined to the body. Thus far, the theory of evolution fits together well. - David, however, has been asking how evolution could have produced various extraordinary symbiotic relationships and methods of entrapment. I would like to extend his question to emotions, consciousness, memory, etc. The "free will" that you are so concerned with, Matt, is another of these faculties ... the result of enhanced consciousness. Like you, I think that we have it, but that we are also greatly influenced by factors (some internal) beyond our conscious control. This leads to two related questions: 
1) Do we believe that all of these astonishingly complex faculties are produced solely by physical cells within the cerebral cortex? 2) If so, do we believe that the relevant sections of the brain ... not to mention the nervous system to which it is connected ... arose initially out of random mutations, i.e. blind chance (though surviving and improving through natural selection)? - If the answer to 1) is yes, then doesn't it mean that we are indeed, as you put it, "slaves to atoms"? If the answer is no, what else is there in us (and, in less developed form, presumably also in other animals) that can produce phenomena such as consciousness, emotion, memory, reason? - If the answer to 2) is yes, so be it. But the greater the complexity, even allowing for millions of years of evolution, the more faith we have to have in chance, which brought about those initial random mutations.

Identity

by xeno6696 @, Sonoran Desert, Tuesday, August 25, 2009, 14:39 (5351 days ago) @ dhw

dhw,
> 
> First, though, another word about animals. There was a sad photograph in this week's Sunday Times, and the caption reads: "Two-year-old orphan Jumaane nudges the face of his dead mother in an attempt to bring her back to life. Yoki, who at 19 was not elderly for an elephant, died on Thursday at the Nyiregyhaza animal park in Hungary, apparently from a tumour in her spleen. Keepers say that since his mother's death, Jumaane has been inconsolable, wandering around aimlessly and weeping."
> 
> There is nothing unusual in this. Elephants not only grieve, but they even stand and contemplate places of death when they return years later. Conversely, they can also celebrate wildly, trumpeting, dancing, and peeing with excitement. If we have common descendants, we should not be surprised that we have common characteristics (which may include the peeing!), and these are not confined to the body. Thus far, the theory of evolution fits together well.
> - Of all the animals worthy of worship by man in his short history, elephants to me have always seemed to be highly esteemed for exactly the reasons you state. (Again, I betray my nym, however.) - > David, however, has been asking how evolution could have produced various extraordinary symbiotic relationships and methods of entrapment. I would like to extend his question to emotions, consciousness, memory, etc. The "free will" that you are so concerned with, Matt, is another of these faculties ... the result of enhanced consciousness. Like you, I think that we have it, but that we are also greatly influenced by factors (some internal) beyond our conscious control. This leads to two related questions: 
> 1) Do we believe that all of these astonishingly complex faculties are produced solely by physical cells within the cerebral cortex? 2) If so, do we believe that the relevant sections of the brain ... not to mention the nervous system to which it is connected ... arose initially out of random mutations, i.e. blind chance (though surviving and improving through natural selection)? 
> 
> If the answer to 1) is yes, then doesn't it mean that we are indeed, as you put it, "slaves to atoms"? If the answer is no, what else is there in us (and, in less developed form, presumably also in other animals) that can produce phenomena such as consciousness, emotion, memory, reason?
> 
> If the answer to 2) is yes, so be it. But the greater the complexity, even allowing for millions of years of evolution, the more faith we have to have in chance, which brought about those initial random mutations. - As much as I bring up Nietzsche, I feel like a parrot, but some of my views were similar to his before I took up the study. This is one of them. - There are some questions that do not have a binary answer. I really wish I could provide even a belief, but with how little wwe actually know in this area, I would feel I'd be putting a lie to my lips by offering even a belief. I mentioned to David, that some of what he was discussing reinvented the "chicken and egg" paradox. - We don't know how this issue arrived, and really at this point in his investigation--it's full of salient and intriguing observations, but my "scientific conservatism" won't allow me to voice a judgment due to the lack of knowledge. I feel you and I become more kin every day, lol. - Clearly, at one time our ancestors did not have the capacity for the kind of thought we have today. They were slaves to their environment, reacting, and never acting. Not until propositional language did human consciousness begin to arise from the murk of the world, giving us agriculture and then everything else. - What I don't understand is why David seems convinced that showing that evolution can happen in short time frames somehow makes a designer more palatable. I also don't see why beautiful and complex animal relationships also do this: symbiosis is explained adequately by selection. Maybe I'm just enough of a materialist that I'm missing an incredible subtlety here, but it just isn't enough to justify the belief in a creator. Just as we don't need to explain the origin of life to use evolution, we don't need to explain how clownfish and anemone became "friends." It doesn't nullify common descent or genetic transmission.

--
\"Why is it, Master, that ascetics fight with ascetics?\"

\"It is, brahmin, because of attachment to views, adherence to views, fixation on views, addiction to views, obsession with views, holding firmly to views that ascetics fight with ascetics.\"

Identity

by David Turell @, Tuesday, August 25, 2009, 19:54 (5351 days ago) @ xeno6696

What I don't understand is why David seems convinced that showing that evolution can happen in short time frames somehow makes a designer more palatable. I also don't see why beautiful and complex animal relationships also do this: . Maybe I'm just enough of a materialist that I'm missing an incredible subtlety here, but it just isn't enough to justify the belief in a creator. Just as we don't need to explain the origin of life to use evolution, we don't need to explain how clownfish and anemone became "friends." It doesn't nullify common descent or genetic transmission. - I don't know what is difficult to understand about my position. Evolution takes time. There are rates of mutation that have been calculated. Most mutations are either bad or neutral. Only some (about 30%) can be useful. Therefore, short time or long time is an important consdieration. I have noted that epigenetic changes can be an important 'short time' mechanism. The ability of RNA to manage genes in DNA is another very important consideration. The more complex the evolutionary machinery, the more it suggests a supernatural origin. Codes come from intelligence. Unless one accepts the Darwin version of DNA, no code ever pops up by chance. - "symbiosis is explained adequately by selection" This declarative statement has no proof.

Identity

by xeno6696 @, Sonoran Desert, Tuesday, August 25, 2009, 22:49 (5351 days ago) @ David Turell

What I don't understand is why David seems convinced that showing that evolution can happen in short time frames somehow makes a designer more palatable. I also don't see why beautiful and complex animal relationships also do this: . Maybe I'm just enough of a materialist that I'm missing an incredible subtlety here, but it just isn't enough to justify the belief in a creator. Just as we don't need to explain the origin of life to use evolution, we don't need to explain how clownfish and anemone became "friends." It doesn't nullify common descent or genetic transmission.
> 
> I don't know what is difficult to understand about my position. Evolution takes time. There are rates of mutation that have been calculated. Most mutations are either bad or neutral. Only some (about 30%) can be useful. Therefore, short time or long time is an important consdieration. I have noted that epigenetic changes can be an important 'short time' mechanism. The ability of RNA to manage genes in DNA is another very important consideration. The more complex the evolutionary machinery, the more it suggests a supernatural origin. Codes come from intelligence. Unless one accepts the Darwin version of DNA, no code ever pops up by chance. 
> - As dhw would likely point out, it would be proof of complexity, but not *proof* of a creator. It's still intangible. It doesn't make it any more *real,* and to me this is because it still doesn't explain anything, because no "how" is generated. To me, for this to be a valid explanation for a deity, you would be required to supply that "how," otherwise it's an explanation that doesn't really explain anything. Proof of *your* abiogenesis has just as big a mountain to climb as the scientific version, in this light. - I've said this before, but for you to claim a creator in this manner, you need to be able to account for both the immaterial and material portions of the creator. You need to be able to exactly define its limits, or you don't have proof, only an interpretation, and like I've railed against often, there is no way to prove an interpretation correct. You read nature like a book and say you see a creator. That's fine, but I have no more reason to believe it over any other interpretation. - > "symbiosis is explained adequately by selection" This declarative statement has no proof. - Natural selection just says that beneficial characteristics would be preserved; to me, that's adequate. I can also use this principle in a lab to produce results. Remember, my metaphysics must do something.

--
\"Why is it, Master, that ascetics fight with ascetics?\"

\"It is, brahmin, because of attachment to views, adherence to views, fixation on views, addiction to views, obsession with views, holding firmly to views that ascetics fight with ascetics.\"

Identity

by David Turell @, Tuesday, August 25, 2009, 23:29 (5351 days ago) @ xeno6696

Unless one accepts the Darwin version of DNA, no code ever pops up by chance. - 
> As dhw would likely point out, it would be proof of complexity, but not *proof* of a creator. 
> Remember, my metaphysics must do something. - I would like you metaphysics tell me how a code appears without intelligence behind it.

Identity

by xeno6696 @, Sonoran Desert, Wednesday, August 26, 2009, 01:43 (5351 days ago) @ David Turell

Unless one accepts the Darwin version of DNA, no code ever pops up by chance. 
> 
> 
> > As dhw would likely point out, it would be proof of complexity, but not *proof* of a creator. 
> > Remember, my metaphysics must do something.
> 
> I would like you metaphysics tell me how a code appears without intelligence behind it. - My metaphysic doesn't require me to answer that question because it recognizes the question as unanswerable to any tool of investigation I possess; even though I do enjoy learning other views such as yours, to me the question is utterly unknowable. My metaphysic is therefore content to leave such questions for ones I *can* investigate. (Note that I separate investigation from speculation.) When I say that natural selection is adequate, it is so because it is the best explanation we can come up with with the available tools, not because it is the most satisfying or complete. 
 
I know you have a lot of passion and thought behind your conviction, but the resistance you receive from outside observers such as myself is ultimately based upon parsimony. To me, any acceptable explanation *must* be studyable. Adding a deity to the model of the universe is... a postscript. What would change about any model assuming a deity exists? What extra thing would it explain? You think a deity would explain the complexity of life, but I say it wouldn't because we still wouldn't be any closer to knowing the how, which is utterly the more important question. We would be no closer to the philosophical and theological question of "why?" It doesn't move anything forward, unless you can enlighten me on some point I'm missing. The only thing I can see it would accomplish is allowing scientists to be less materialistic, hopefully getting more theists into the fold. - Look, you've told me that your view is essentially panentheistic, and of course I can respect that, but unless you can tell me how this deity works, you're in no better position than those who support scientific abiogenesis. In fact in a great many ways you're at a disadvantage in terms of explanation, as there is no way to determine "good" theological explanations from "bad." - This is why I've liked the process philosophy that you seem to dislike, because it at least attempts to provide *some* kind of explanation for God's interface with the world.

--
\"Why is it, Master, that ascetics fight with ascetics?\"

\"It is, brahmin, because of attachment to views, adherence to views, fixation on views, addiction to views, obsession with views, holding firmly to views that ascetics fight with ascetics.\"

Identity

by David Turell @, Wednesday, August 26, 2009, 16:51 (5350 days ago) @ xeno6696

My metaphysic doesn't require me to answer that question because it recognizes the question as unanswerable to any tool of investigation I possess; - Matt, I appreciate your long response. It helps me define the limits of your thinking. We have come to the 'how' desire and the 'why' desire, and that is where we have separated. I've been looking at 'how' life works for many years, and I find it amazing, and beyond belief that it popped up from many chance contingencies. 
 
> I know you have a lot of passion and thought behind your conviction, but the resistance you receive from outside observers such as myself is ultimately based upon parsimony. - I think it is less parsimonious to require all those steps of contingency, than to assume a supernatural force. A supernatrual force, if present, requires that we accept the fact that we will never know the 'how', completely. In his book, A Concealed God, Stefan Einhorn, a philosopher of religion,states: "Science has not yet been and may never be able to provide all the answers. Taking the matter to its extreme, we can say there are two (not mutually exclusive) explanatory models. Either God is a biochemical process in the human brain, the function of which is to protect the intellect from experiencing the world as insecure and meaningless--feelings that could have resulted in the downfall of the human race in evolutionary perspective--or else there is a God." - 
> Look, you've told me that your view is essentially panentheistic, and of course I can respect that, but unless you can tell me how this deity works, you're in no better position than those who support scientific abiogenesis. In fact in a great many ways you're at a disadvantage in terms of explanation, as there is no way to determine "good" theological explanations from "bad." - Einhorn, above, is an answer to this comment of yours, but I have a further thought. My approach is to expect science to continue to demonstrate increasing complexity in the coded management of life. I have indicated overtures to this complexity in past entries, briefly, epigenetic mechanisms, and RNA management of genes. Watson-Crick discovery of the coding of protein molecules is a very simplistic beginning to what is now being discovered in the management of those proteins. They just put the camel's head into the tent. My expected 'proof of God' will be exclusionary. The complexity of life will be found to be so enormous, only something supernatural can have created it. We will never fully know the 'why', because we cannot. And so a faith takes over. No one can use 'how' to get to God. Pascal's leap will always be required. For me I've taken the leap, and it feels good, just as my love for my wife feels good. Yes, this is at an emotional level, but at some point that level is required. - I was invited to join this website. I've acted as a gadfly and will continue to do so. I do understand everyone's position, and will try to continue to have everyone defend them.

Identity

by xeno6696 @, Sonoran Desert, Wednesday, August 26, 2009, 20:30 (5350 days ago) @ David Turell

My metaphysic doesn't require me to answer that question because it recognizes the question as unanswerable to any tool of investigation I possess;
> 
> Matt, I appreciate your long response. It helps me define the limits of your thinking. We have come to the 'how' desire and the 'why' desire, and that is where we have separated. I've been looking at 'how' life works for many years, and I find it amazing, and beyond belief that it popped up from many chance contingencies. 
> - Yeah, I've suspected for some time that the differences we had really had to be "perspectivist" in nature, and it helps me *immensely* to know the broader foundation of your beliefs--something perhaps I should have just directly asked for before. [EDIT] But now at least I understand it--thank you. - 
> > I know you have a lot of passion and thought behind your conviction, but the resistance you receive from outside observers such as myself is ultimately based upon parsimony. 
> 
> I think it is less parsimonious to require all those steps of contingency, than to assume a supernatural force. A supernatrual force, if present, requires that we accept the fact that we will never know the 'how', completely. In his book, A Concealed God, Stefan Einhorn, a philosopher of religion,states: "Science has not yet been and may never be able to provide all the answers. Taking the matter to its extreme, we can say there are two (not mutually exclusive) explanatory models. Either God is a biochemical process in the human brain, the function of which is to protect the intellect from experiencing the world as insecure and meaningless--feelings that could have resulted in the downfall of the human race in evolutionary perspective--or else there is a God."
> - I guess I'm not sure about the contingency unless you mean scientific models at large? - It was Nietzsche that got me up off my ass and made me "mystical" in whatever strange form its taken in me. I can say for the first time at least that I know what it means to be spiritual. I've learned more from the "higher thinking" of philosophers and theologians than I necessarily have from my scientific training. - > 
> > Look, you've told me that your view is essentially panentheistic, and of course I can respect that, but unless you can tell me how this deity works, you're in no better position than those who support scientific abiogenesis. In fact in a great many ways you're at a disadvantage in terms of explanation, as there is no way to determine "good" theological explanations from "bad." 
> 
> Einhorn, above, is an answer to this comment of yours, but I have a further thought. My approach is to expect science to continue to demonstrate increasing complexity in the coded management of life. I have indicated overtures to this complexity in past entries, briefly, epigenetic mechanisms, and RNA management of genes. Watson-Crick discovery of the coding of protein molecules is a very simplistic beginning to what is now being discovered in the management of those proteins. They just put the camel's head into the tent. My expected 'proof of God' will be exclusionary. The complexity of life will be found to be so enormous, only something supernatural can have created it. We will never fully know the 'why', because we cannot. And so a faith takes over. No one can use 'how' to get to God. Pascal's leap will always be required. For me I've taken the leap, and it feels good, just as my love for my wife feels good. Yes, this is at an emotional level, but at some point that level is required.
> 
> I was invited to join this website. I've acted as a gadfly and will continue to do so. I do understand everyone's position, and will try to continue to have everyone defend them. - Don't stop. I'm at this website because I continue searching, not that I believe I have found an answer. Too much of my persona fights against the concept of faith... even as a child I never liked it. I also know I only grow when pushed, so keep pushing, and I'll keep growing. (Goes for anyone else who reads this, btw.) - When I was a teenager, I got into a minor religious conversation with a buddy at work. His answer seemed comical to me then, he was incredulous at the time that I called myself an atheist, and when asked why he even cared he said "I think that each view has a small portion of the truth." I recognize that as true wisdom in my "ripe age" at present. Sucks being a late-bloomer, lol. - When reading Emerson, I know exactly what Nietzsche's ubermensch was and why it was unattainable by any man. Each person and their singularly defined virtue makes up the ubermensch. [EDIT] The same goes for our collected wisdom.

--
\"Why is it, Master, that ascetics fight with ascetics?\"

\"It is, brahmin, because of attachment to views, adherence to views, fixation on views, addiction to views, obsession with views, holding firmly to views that ascetics fight with ascetics.\"

Identity

by David Turell @, Thursday, August 27, 2009, 00:29 (5350 days ago) @ xeno6696

I guess I'm not sure about the contingency unless you mean scientific models at large? - 
No, I'm using Gould's terminology. If everything is a chance occurance in evolution, then each step is contingent upon the last one. - 
> Don't stop. I'm at this website because I continue searching, - I'm not stopping. This is fun and I have a point of view I can defend. - 
> and when asked why he even cared he said "I think that each view has a small portion of the truth." I recognize that as true wisdom in my "ripe age" at present. Sucks being a late-bloomer, lol. - It is a matter of age. At your age I was definitely agnostic. The more I practiced medicine, the more I decided I had to do some outside reading and make up my mind. Medicine demonstrates too many miracles. I've never been moderate. Sitting on the picket fence is too hard on the butt!

Identity

by xeno6696 @, Sonoran Desert, Thursday, August 27, 2009, 04:30 (5350 days ago) @ David Turell

I guess I'm not sure about the contingency unless you mean scientific models at large? 
> 
> 
> No, I'm using Gould's terminology. If everything is a chance occurance in evolution, then each step is contingent upon the last one. 
> - I guess I don't see a problem with that. But don't you think statistically you might also be committing a similar error to... I don't remember who argued this, but say you take a random rock in your backyard. What are the chances that rock is in your backyard? For that answer, you take it's surface area--to make the math simple we'll say its 1m x 1m--and do a combination by the total surface area on earth, 510,072,000km2. - The odds of that rock being in your backyard are incredibly awful, but it doesn't change the fact that it's there. And that rock being in your backyard had to have a same contingency of events to make it to your backyard as well. We could make the number even smaller by taking into consideration the type of rock, its distance from your home, whether or not it was brought there by another person, you name it. - To me it seems you do exactly the same thing with your argument about evolution. You ask a series of probability questions--when (no offense) the questions might be irrelevant. There is no special meaning behind the rock in your backyard, and similarly I can fathom nothing from life. (Mayhaps I'm just too jaded?) - > 
> > Don't stop. I'm at this website because I continue searching, 
> 
> I'm not stopping. This is fun and I have a point of view I can defend.
> 
> 
> > and when asked why he even cared he said "I think that each view has a small portion of the truth." I recognize that as true wisdom in my "ripe age" at present. Sucks being a late-bloomer, lol. 
> 
> It is a matter of age. At your age I was definitely agnostic. The more I practiced medicine, the more I decided I had to do some outside reading and make up my mind. Medicine demonstrates too many miracles. I've never been moderate. Sitting on the picket fence is too hard on the butt! - I probably won't develop similarly... computer science has no miracles, lol. - My time in medicine has been dismal, though working in a trauma center will give you few if any miracles. Once, and only once I was present for a 26yr old female who they rolled straight to the OR, bad enough that they had her chest cracked and were massaging it on route. She got a pulse back... - Every person cheered, the one of the doctors breathed a huge sigh of relief. - ...for approximately thirty seconds. - This year was the first year I got angry about it. I've worked 250+ trauma nights. Not once have I seen a miracle. It's bad enough that I can tell you in about one or two minutes whether or not I'm going to be working. This is the kind of life experience thing I'm talking about--no more can I really say anything about yours, there isn't really much you can say about mine, it's luck of the draw in medicine.

--
\"Why is it, Master, that ascetics fight with ascetics?\"

\"It is, brahmin, because of attachment to views, adherence to views, fixation on views, addiction to views, obsession with views, holding firmly to views that ascetics fight with ascetics.\"

Identity

by dhw, Wednesday, August 26, 2009, 11:30 (5350 days ago) @ xeno6696

Matt cannot answer the questions I asked at the end of my post of Tuesday 25 August at 07.55. Nor of course can I, though I think this discussion still has a long way to run. - You write "There are some questions that do not have a binary answer. I really wish I could provide even a belief, but with how little we actually know in this area, I would feel I would be putting a lie to my lips by offering even a belief. [...] I feel you and I become more kin every day." - We are indeed in exactly the same position in this respect (as I think we are in many other areas too, including elephant territory). However, there is a subtle difference between us. You don't understand why David's view of evolution and symbiosis "somehow makes a designer more palatable". David has already answered for himself, and I have posted part of my answer on the "Nature's IQ" thread. I take each of these "Nature's IQ" and "Evolution" posts only as a section and not as the whole of the argument. The basis of it all, as we have repeated ad nauseam, is the complexity of life, and so I can understand perfectly well why someone would find the evidence for design overwhelming enough "to justify the belief in a creator". Where one goes from there is another matter, and David has his own (perhaps somewhat unconventional) views on such a being. However, I can also understand perfectly well that the very concept of a designer raises insoluble problems (see "Arguments Against Design", 29 July at 13.13). I remain open-minded/on the fence/pressing the pause button/hopelessly indecisive, and therefore absolutely not prepared to dismiss people's personal experiences as possible evidence, whereas I think you are more inclined towards the materialist view because of what you call your "scientific conservatism".

Identity

by xeno6696 @, Sonoran Desert, Wednesday, August 26, 2009, 18:18 (5350 days ago) @ dhw

Matt cannot answer the questions I asked at the end of my post of Tuesday 25 August at 07.55. Nor of course can I, though I think this discussion still has a long way to run. 
> 
> You write "There are some questions that do not have a binary answer. I really wish I could provide even a belief, but with how little we actually know in this area, I would feel I would be putting a lie to my lips by offering even a belief. [...] I feel you and I become more kin every day."
> 
> We are indeed in exactly the same position in this respect (as I think we are in many other areas too, including elephant territory). However, there is a subtle difference between us. You don't understand why David's view of evolution and symbiosis "somehow makes a designer more palatable". David has already answered for himself, and I have posted part of my answer on the "Nature's IQ" thread. I take each of these "Nature's IQ" and "Evolution" posts only as a section and not as the whole of the argument. The basis of it all, as we have repeated ad nauseam, is the complexity of life, and so I can understand perfectly well why someone would find the evidence for design overwhelming enough "to justify the belief in a creator". Where one goes from there is another matter, and David has his own (perhaps somewhat unconventional) views on such a being. However, I can also understand perfectly well that the very concept of a designer raises insoluble problems (see "Arguments Against Design", 29 July at 13.13). I remain open-minded/on the fence/pressing the pause button/hopelessly indecisive, and therefore absolutely not prepared to dismiss people's personal experiences as possible evidence, whereas I think you are more inclined towards the materialist view because of what you call your "scientific conservatism". - Resigned to that conservatism might be a better way to say it. - There's two prongs to why I don't consider individual experience in *this* question. - When it comes to other people's individual experiences, such as your discussion about the juju, I do not like to work with them because it is sacred territory. A person's experiences are inviolable. Your experiences are not open to study by me, because there is no way to recreate the event except in words. One of my best friends became Christian when he encountered what he is convinced were demons and he was able to scatter them by calling the name of Jesus for help. I cannot feasibly debate or attack an issue like that without necessarily attacking my friend. This goes for you and David as well; I don't view it as wearing "kid gloves" but the "humanity" of such events always writes an indelible ink on our soul and it is impossible to remove the self from them. - The second issue is that I would need to have been present to offer my own analysis of the event in question... the act of recounting an event from memory is itself always worth casting doubt on. My wife for example, always remembers things differently from me, and police officers have noted that witness testimonies to car crashes rarely if ever corroborate. I myself have been on the receiving end of this exact phenomenon, having given testimony completely different from 2 other witnesses and one of the drivers. - This means that the level of subjectivity is simply too high for my analytical taste. When you couple subjectivity to events of a personal nature... it is my experience that no real movement ever happens in such discussions, and more often then not the result is ad hominem and anger.

--
\"Why is it, Master, that ascetics fight with ascetics?\"

\"It is, brahmin, because of attachment to views, adherence to views, fixation on views, addiction to views, obsession with views, holding firmly to views that ascetics fight with ascetics.\"

Identity

by dhw, Friday, August 28, 2009, 08:53 (5348 days ago) @ xeno6696

I wrote that I was "absolutely not prepared to dismiss people's personal experiences as possible evidence". Matt replied: "A person's experiences are inviolable. Your experiences are not open to study by me, because there is no way to recreate the event except in words." And also: "The act of recounting an event from memory is itself always worth casting doubt on. [...] This means that the level of subjectivity is simply too high for my analytical taste." - In your response to David (26 August at 01.43) you also wrote "To me, any acceptable explanation must be studyable." - All things are "studyable", but I suspect that what you mean by that is scientifically testable. It comes down to the usual question of whether you do or don't believe that science holds all the answers ... and that in turn comes down to whether you are or are not a materialist. (I'm aware that you are not, but when asked to lean, you tend to go in that direction.) "Subjectivity" is a dirty word in scientific contexts, but I would say that subjective experiences provide us with most of the realities that make life worth living. I hope you didn't insist on a scientific study of the cerebral cortices of your wife and yourself before you told her you loved her. - Subjective does not mean unreal or untrue ... and I know you're not saying it does ... but I recall in one of your earlier posts that you dismissed all "paranormal" experiences as fake. (Forgive me if that was not the precise word ... I'm too lazy to go hunting for it ... but it was certainly something like that.). You're right to avoid personal conflict in individual cases, and in general it seems to me quite fair to suspend judgement. But without knowing the facts, it's unfair to pass judgement, let alone impose prejudgement. I wrote that I was absolutely not prepared to dismiss people's personal experiences as possible evidence. Your reply suggests, in the nicest possible way, that you are.
 
I would add, though, that we're ALL subject to personal experiences we don't understand and can't analyse objectively, but because we're so used to them, we're largely unaware of them. That's the subject I'm trying to grapple with under this thread, but it's very difficult to articulate. I shall have another go at it in due course. Your inability and my own to answer the questions I posed on 25 August at 07.55 lie at the heart of it.

Identity

by David Turell @, Friday, August 28, 2009, 14:59 (5348 days ago) @ dhw

and that in turn comes down to whether you are or are not a materialist. (I'm aware that you are not, but when asked to lean, you tend to go in that direction.) "Subjectivity" is a dirty word in scientific contexts, but I would say that subjective experiences provide us with most of the realities that make life worth living.
> Subjective does not mean unreal or untrue ... and I know you're not saying it does ... but I recall in one of your earlier posts that you dismissed all "paranormal" experiences as fake......> I wrote that I was absolutely not prepared to dismiss people's personal experiences as possible evidence. Your reply suggests, in the nicest possible way, that you are.
> 
> I would add, though, that we're ALL subject to personal experiences we don't understand and can't analyse objectively, but because we're so used to them, we're largely unaware of them. That's the subject I'm trying to grapple with under this thread, but it's very difficult to articulate. - At Uncommon Descent, second entry today has an interesting article by William Lycan discussing materialism vs. Cartesian dualism that applies to the above discussion: www.uncommondescent.com . It is at the word SOURCE.

Identity

by dhw, Saturday, August 29, 2009, 11:53 (5347 days ago) @ David Turell

David has recommended an article by William G. Lycan on materialism vs. Cartesian dualism: www.uncommondescent.com, to be located at the word SOURCE. - Lycan is a self-confessed materialist on this subject, but the essay is extremely helpful and beautifully balanced. I hope everyone will read it. Here are two sample quotes to illustrate the fairness of his approach: - "To anyone uncontaminated by neuroscience or materialist philosophizing, the mental does not seem physical in any way at all, much less neurophysiological. The parsimony argument does not even come in the door until it is agreed that we can find nothing to distinguish mental states from neurophysiological ones. And the latter will not be agreed any time soon." - "I mean to have shown here that although Cartesian dualism faces some serious objections, that does not distinguish it from other philosophical theories, and the objections are not an order of magnitude worse than those confronting materialism in particular." - Once more, many thanks to David for this reference.

Identity

by George Jelliss ⌂ @, Crewe, Saturday, August 29, 2009, 16:45 (5347 days ago) @ dhw

As far as I'm concerned ones mind is the activity of ones brain. This is a form of "dualism" since it says that mind and brain are different concepts, but it is also a form of "materialism", since ones brain is composed of matter. I really don't see that there is any philosophical problem here at all. If there is any problem it is just in our primitive knowledge of neuroscience, that is of exactly how our brains work. But I suppose that's what makes it a "philosophical" problem as opposed to a real one!

--
GPJ

Identity

by dhw, Sunday, August 30, 2009, 08:35 (5346 days ago) @ George Jelliss

George: As far as I'm concerned ones mind is the activity of ones brain. This is a form of "dualism" since it says that mind and brain are different concepts, but it is also a form of "materialism", since ones brain is composed of matter. I really don't see that there is any philosophical problem here at all. If there is any problem it is just in our primitive knowledge of neuroscience, that is of exactly how our brains work. But I suppose that's what makes it a "philosophical" problem as opposed to a real one! - The fact that you attribute the "problem" to our primitive knowledge of neuroscience simply means that you believe you know the answer before it has been found. You may be right, of course, but you have no more evidence for your belief than those who think that our minds are part of a universal intelligence.

Identity

by George Jelliss ⌂ @, Crewe, Sunday, August 30, 2009, 09:38 (5346 days ago) @ dhw

George: As far as I'm concerned ones mind is the activity of ones brain. This is a form of "dualism" since it says that mind and brain are different concepts, but it is also a form of "materialism", since ones brain is composed of matter. I really don't see that there is any philosophical problem here at all. If there is any problem it is just in our primitive knowledge of neuroscience, that is of exactly how our brains work. But I suppose that's what makes it a "philosophical" problem as opposed to a real one!
> 
> The fact that you attribute the "problem" to our primitive knowledge of neuroscience simply means that you believe you know the answer before it has been found. You may be right, of course, but you have no more evidence for your belief than those who think that our minds are part of a universal intelligence. - This is the problem I have all the time in attempting to argue on this forum. The only reliable evidence we have is that we are made of atoms. The evidence for this is overwhelming. Yet you disregard this evidence completely! There is no evidence for "universal intelligence" or disembodied minds other than subjective speculation and fanciful imagination. Perhaps we are all part of some "Matrix" style plot, or part of the Red King's dream. Perhaps we are just programs in some supercomputer. There is no end to such imaginative speculations. My views are solidly based on what we actually know.

--
GPJ

Identity

by David Turell @, Sunday, August 30, 2009, 18:53 (5346 days ago) @ George Jelliss

This is the problem I have all the time in attempting to argue on this forum. The only reliable evidence we have is that we are made of atoms. The evidence for this is overwhelming. Yet you disregard this evidence completely! There is no evidence for "universal intelligence" or disembodied minds other than subjective speculation and fanciful imagination. - This should not be a problem. I agree there is no evidence for a universal intelligence, but it is the best supposition I can come up with as an answer for me. From the evidence, as I interpret it, chance has no chance at all for creating what we have as our life and reality. You choose a different conclusion, as is your right. I don't know how much background you have in biochemistry (the chemistry of life) and mammalian physiology. but what I have seen is close to miraculous. For example, in kidney dialysis, our best efforts don't come close to what the kidney really does. Frankly, we do a half-baked job which is barely adequate, which is why transplant is the preferred method. We still don't have a good way of substituting for the liver, which is a much more complex organ in its functions. Heart transplants work so well because it is a simple muscular pump, not very complex at all. I will ask a direct question. How much of this do you understand? Because it is the key to my point of view.

Identity

by George Jelliss ⌂ @, Crewe, Sunday, August 30, 2009, 21:53 (5346 days ago) @ David Turell

DT wrote: "I agree there is no evidence for a universal intelligence, but it is the best supposition I can come up with as an answer for me." - But what is it a supposition of? What does it actually consist of? How does it work? As I understand it you think your mind is a part of this universal intelligence? But you don't really have any practical theory about how it actually works. It's all so very vague. - What we do actually know is that our brains are made of atoms and that they work by means of electrochemical connections between the millions of cells, and it is this activity that we experience as mind. How does your hypothesis of disembodied mind work in this context? - I'm quite convinced that the complexities of biology are adequately explained to have arisen through evolution by natural selection, or "by chance" as you insist on calling it. I agree that the results are "close to miraculous". But we've already argued that issue ad nauseam. - DT asked: "I will ask a direct question. How much of this do you understand? Because it is the key to my point of view." I'm not a biologist, but I've read a lot and like to keep up to date. My background is in maths, physics and chemistry. I used to work in science publishing.

--
GPJ

Identity

by David Turell @, Tuesday, September 01, 2009, 01:49 (5345 days ago) @ George Jelliss

But what is it a supposition of? What does it actually consist of? How does it work? As I understand it you think your mind is a part of this universal intelligence? But you don't really have any practical theory about how it actually works. -No I don't, and you understand this beforehand. I am thinking in the realm of the supernatural. I don't have any exact idea any more than the monotheistic religions have of their Gods. The Eastern religions can't offer any help. There is no way to offer a 'practical theory' when materialistic reductionism in our current scientific approach cannot study the supernatural. If all of us could directly analyze possible divine levels, we would not be having the current discussions on this website. It is simply beyond my willingness to accept natural causes for something that is as complex as the functions of our bodies and the development of our conscious minds. Further I have never seen a complex code that did not have an intellectual source. A universal intelligence is about as far as I can go in developing a concept I can feel comfortable with. Can my thought be proven? Of course not. But if science continues to expand our knowledge of the complexity of life, I think we will reach a point (a probability bound) that mathematical probabilities will require that we accept something supernatural, 
 
> What we do actually know is that our brains are made of atoms and that they work by means of electrochemical connections between the millions of cells, and it is this activity that we experience as mind. How does your hypothesis of disembodied mind work in this context?-Our brain is made up of billion of neurons with trillions of synaptic connections. Part of this exists as we are born, but the brain develops much of it as we learn and develop memories. PET scans show where much of the functions occur, but there are so many minor interconnections, I'm not sure the scans give a perfect picture to give us full understanding. I do think that our consciousness arises at a quantum level of electrical activity and is an emergent phenomenon, raising the old debate about dualism. Part of my thinking is that the universal intelligence is also at the quantum reality level, again, as I have stated before, across a wall of uncertainty (Heisenberg) that we are unable to explore, but that intelligence somehow is connected to our small intelligence in a quantum fashion.- This allows for my thinking that there are paranormal psychic experiences. Collections of near-to-death experiences find that they are very similar; they always see dead people if people are seen or communicated with; some NDEer's learn about the death of others during the experience, without any way of that information coming to them other than the experience. These have been documented generally in hospice settings. My wife is occasionally psychic. She does not control or ask for the 'visions'. 
 
> I'm quite convinced that the complexities of biology are adequately explained to have arisen through evolution by natural selection, or "by chance" as you insist on calling it. -I believe that evolution occurred. Natural selection is another term for competition between species and/or individuals in the same species. Mutations occur at random and occasionally change an individual to improve his chances for survival. Mutations can change species to improve their survival. According to Darwin, evolution is undirected because mutations are random. Mutations, therefore, are chance events. Natural selection is a competition that can occur only between differing individuals with different capacities for survival. Mutations, as chance events, are a passive process. Natural selection, as a competition, is an active process, but since the starting event is passive, evolution must be considered a passive activity. That is why I use the word 'chance'. -> DT asked: "I will ask a direct question. How much of this do you understand? Because it is the key to my point of view." I'm not a biologist, but I've read a lot and like to keep up to date. My background is in maths, physics and chemistry. I used to work in science publishing.-Thank you for the information. It helps me understand some of your areas of advanced knowledge, and perhaps may help me understand why you think the way you do

Identity

by dhw, Monday, August 31, 2009, 11:33 (5345 days ago) @ George Jelliss

George: This is the problem I have all the time in attempting to argue on this forum. The only reliable evidence we have is that we are made of atoms. The evidence for this is overwhelming. Yet you disregard this evidence completely! There is no evidence for "universal intelligence" or disembodied minds other than subjective speculation and fanciful imagination, [...] My views are solidly based on what we actually know.-I agree with you that there is no evidence for "universal intelligence". My point was that you had no more evidence for your materialism than those who think our minds are part of such an intelligence. The problem for me is that you don't seem to acknowledge that there IS a problem. No-one is denying that we are made of atoms, and no-one is denying that all our mental activities are somehow connected with our physical brains which work "by means of electrochemical connections" (your latest post). But despite all the progress of neuroscience, no-one has yet explained how these blobs of matter can actually produce thought, emotion, imagination, and consciousness of themselves. If I decide to do a crossword puzzle, are my brain cells telling me what to do, or am I telling my brain cells what to do, and if my mind is made only of brain cells, what constitutes "I/me"?-There are two kinds of thoughts: those that come to us involuntarily, and those which we form or process deliberately. In the first instance, why and how do my brain cells confront "me" with ideas, memories, imaginings, independently of my will (another product of my brain cells)? In the second, how does my brain-cell-will control the processes of my brain-cell-thinking? Furthermore, if my identity is fashioned by heredity, environment, chance and experience, but my mind (which encompasses my identity) consists only of brain cells, how are the brain cells changed by environment, chance and experience? Do I have any control over such changes ... and if so, what gives me that control? -My mind does not SEEM to me to consist solely of brain cells ... because I feel that I have some control over my brain ... but you tell me it does, and you say your views are solidly based on what we actually know. But we do not "actually know" anything about how all the above mental processes work. My own views are CONFINED to what we actually know, and therefore until a satisfactory physical explanation of these processes is found, I simply cannot say that the mind is nothing but atoms. And so long as that question remains open, I cannot say that there are no dimensions or forms of existence beyond those of the physical world as we now know it.

Identity

by George Jelliss ⌂ @, Crewe, Monday, August 31, 2009, 19:42 (5345 days ago) @ dhw

dhw wrote: "I agree with you that there is no evidence for "universal intelligence"."-A surprising admission!-dhw wrote: "If I decide to do a crossword puzzle, are my brain cells telling me what to do," No. "or am I telling my brain cells what to do," No. "and if my mind is made only of brain cells," No. "what constitutes "I/me"?" See my earlier post where I indicated that we are our past history. You seem to agree with this when you say: "Furthermore, if my identity is fashioned by heredity, environment, chance and experience, ..."-but then you go on: "... but my mind (which encompasses my identity) consists only of brain cells," but I've specifically stated that the mind is the activity of the brain, not the brain cells themselves.-dhw: "how are the brain cells changed by environment, chance and experience?" -It is the connections between them that are changed, and what they are taken to signify. 
 
dhw: "My mind does not SEEM to me to consist solely of brain cells" -I never said it did consist of brain cells.-dhw: "But we do not "actually know" anything about how all the above mental processes work." -We do know a great deal. What do you thnk all those neuroscientists and psychologists are doing with their time?-dhw: "I simply cannot say that the mind is nothing but atoms." -Same old straw man again!

--
GPJ

Identity

by David Turell @, Tuesday, September 01, 2009, 01:53 (5345 days ago) @ George Jelliss


> dhw: "I simply cannot say that the mind is nothing but atoms." 
> 
> Same old straw man again!-The brain is nothing but atoms. The mind is another issue. It is created by those atoms, but how? See my last post.

Identity

by dhw, Tuesday, September 01, 2009, 20:47 (5344 days ago) @ George Jelliss

George: I've specifically stated that the mind is the activity of the brain, not the brain cells themselves.-Fair comment, but this only shifts all my questions from "brain cells" to "activity". With involuntary thought, what causes the brain-cell activity that results in ideas, memories, imaginings? How does this activity translate itself into such ideas, memories, imaginings? 
With deliberate thought, what gives us control over the activity? -As regards identity, it's still the same problem. You say it's the connections between the brain cells that are changed, and "what they are taken to signify". We agree that changes to our identity are brought about by environment, chance and experience (which you sum up as our history ... OK by me), but I'm asking for explanations no-one has yet provided: HOW do my atoms change their own connections? Do "I" have any control over this process, and if so HOW do my atoms give "me" this control? When you say: "What they are taken to signify" ... what does the taking? -Strikingly, you asked David how a "universal intelligence" actually works. I am asking the same question about the activities of the brain.-I wrote: But we do not know anything about how all the above mental processes work. 
You responded: We know a great deal. What do you think all those neuroscientists and psychologists are doing with their time?-I was referring only to the mechanisms (sorry, I know you hate the word) that provide us with thought, will, memory, emotion, consciousness, imagination etc. ... not to psychology. Neuroscientists can tell us which sections of the brain are connected with which activity, but nobody knows HOW the activities translate themselves into these faculties.-I wrote: I simply cannot say that the mind is nothing but atoms.
You wrote: Same old straw man again!
 
I can do no better than quote David Turell's response: "The brain is nothing but atoms. The mind is another issue. It is created by those atoms, but how?" I'm not sure if "created" is the right word, but since I don't understand the process, I don't know what IS the right word. All the same, let me apply the conclusion of my last post to David's response: "And so long as that question remains open, I cannot say that there are no dimensions or forms of existence beyond those of the physical world as we know it." Your belief in the creative powers of our atoms (and their activities) is strong enough for you to dispense with any concept beyond the physical, and that of course is your prerogative. I can only tell you why I don't share your belief.-However, not sharing one belief does not mean espousing another. You began your post by expressing surprise at my agreement that there is no evidence for a "universal intelligence". I'm surprised that you're surprised. Perhaps you've forgotten that I'm an agnostic!

Identity

by George Jelliss ⌂ @, Crewe, Thursday, September 03, 2009, 00:03 (5343 days ago) @ dhw

I'm finding it increasingly difficult to follow this forum. The messages no longer seem to follow a straightforward sequence, but dodge bout all over the place.-dhw: "I'm asking for explanations no-one has yet provided: HOW do my atoms change their own connections?" The connections are changed by our experiences.-dhw: "Do "I" have any control over this process," Yes, you can seek out experiences, and you can associate different thoughts.-dhw: "HOW do my atoms give "me" this control?" You are the history of your experiences. -dhw: "When you say: "What they are taken to signify" ... what does the taking?" A connection between brain cells comes about as a result of an experience and thus represents that experience. It is a memory of that experience.
 
dhw: "You began your post by expressing surprise at my agreement that there is no evidence for a "universal intelligence". I'm surprised that you're surprised. Perhaps you've forgotten that I'm an agnostic!"-If you agree there is no evidence for such an entity, why do you bother giving it credence?

--
GPJ

Identity

by David Turell @, Thursday, September 03, 2009, 02:00 (5343 days ago) @ George Jelliss

If you agree there is no evidence for such an entity, why do you bother giving it credence?-There can never be direct evidence. All of us have to accept that fact. However, I still maintain, if science discovers an increasingly staggering amount of coding complexity (DNA/RNA)to run living organisms, very minute odds for a series of chance contingencies will appear pointing to the acceptance that chance could not have created this code. It is sort of a negative proof, but can reach a probability bound that will require the reasonable decision that something supernatural is going on. To understand this prediction, one must accept my definition of Darwinian evolution as passive. If it is purposeless, it is passive. If mutations are chance events, it is passive. If the number of contingent events required are 10 to the 100th power, the supernatural becomes the best bet.-I agree we have been jumping around. But issues that must be covered include cosmology, particle physics, quantum theory, the validity of the Darwin Theory, etc. I know you know this, but to my mind interesting findings pop up in many of these areas, and if I find something that I think applies, I think it should be exhibited for all to review. If you mind my jumping in to your conversations with dhw, I apologize. Just let me know.

Identity

by dhw, Friday, September 04, 2009, 08:20 (5341 days ago) @ George Jelliss

George is finding it increasingly difficult to follow this forum, and I must confess I'm finding it increasingly difficult to follow George's replies, because they do not relate to my questions! For instance:-1) dhw: HOW do my atoms change their own connections?
George: The connections are changed by our experiences.
Your answer tells me WHAT changes the connections. I do not understand HOW the fact that I have an experience enables my atoms to change their connections. The atoms are physical. I might be a miser who sees the ghosts of Christmas past, present and future, and realizes that meanness doesn't bring happiness. How will this alter the physical connections between my atoms?-2) dhw: HOW do my atoms give "me" this control?
George: You are the history of your experiences.
Your answer does not tell me how my physical atoms create control over the way in which my physical atoms connect themselves.-In all your answers, there is a dimension missing: namely, the link between the activities of the physical atoms and the (for want of a better word) abstract identity. You seem to take the latter for granted. When I asked, "Do "I" have control over this process?" [i.e. the atoms changing their own connections], you answered: "Yes, you can seek out experiences, and you can associate different thoughts." But my basic questions are: what constitutes this "you" and how, physically, does it control and change itself? According to your answer to 2), the history of my experiences seeks out experiences and associates different thoughts, which doesn't help much.-As I see it, my identity is composed of my thoughts, attitudes, emotions, imagination, memories etc., some of which are involuntary (out of my control) and some of which are deliberate. All of them are encompassed within my mind. If my mind is the "activity of the brain", which means the activity set in motion by my physical atoms, I want to know what causes the involuntary activities (e.g. a new idea), what enables me to guide the deliberate ones (i.e. what actually endows me with consciousness and will), and what translates the "electrochemical connections" into thoughts, attitudes etc. -The materialist view is that there has to be a physical explanation, but nobody knows it (yet). The non-materialist view is that we have a non-physical identity, which some call the soul and some say is part of a universal intelligence. There is no evidence for this. So there we have two theories, neither of which has any known basis. You ask, "If there is no evidence for such an entity [universal intelligence], why do you bother giving it credence?" I don't. Nor do I give credence to the materialist theory. Not believing is one side of agnosticism. The other side is not disbelieving.-David apologizes for jumping into this conversation. I wish more people would do so. None of these discussions are private, and since George and I are having such difficulty communicating, it would be of great help to have some more opinions.

Identity

by George Jelliss ⌂ @, Crewe, Friday, September 04, 2009, 10:50 (5341 days ago) @ dhw

dhw: "I'm finding it increasingly difficult to follow George's replies, because they do not relate to my questions! For instance:-1) dhw: HOW do my atoms change their own connections?
George: The connections are changed by our experiences.
/// I do not understand HOW the fact that I have an experience enables my atoms to change their connections. ///
 
2) dhw: HOW do my atoms give "me" this control?
George: You are the history of your experiences.
Your answer does not tell me how my physical atoms create control over the way in which my physical atoms connect themselves."-
You need to read up on the science that has been done. It is diffcult to describe what happens in brief sentences that use only broad philosophical concepts. This may be helpful (there are preceding pages too that give a broader introduction):-http://www.web-us.com/brain/bio_org.html-I'm sure there are many other sites on this subject. This is just one that I have found gives a clear exposition.-Nature has a discussion board on Brain physiology, cognition and consciousness:-http://network.nature.com/groups/bpcc-
dhw: "The materialist view is that there has to be a physical explanation, but nobody knows it (yet)."-But we do know it to a considerable extent! You need to update your knowledge. We've come a long way since Descartes!-dhw: "The non-materialist view is that we have a non-physical identity, which some call the soul and some say is part of a universal intelligence. There is no evidence for this. So there we have two theories, neither of which has any known basis."-The "materialist" view as you call it, I would prefer to call it something like the "scientific" view, does have a very sound basis.-dhw: "Not believing is one side of agnosticism. The other side is not disbelieving." Exactly, but you need to proportion your degree of belief to the degree of evidence! Just because two different views of a subject can be presented does not mean that they are equally valid, 50:50.

--
GPJ

Identity

by xeno6696 @, Sonoran Desert, Saturday, September 05, 2009, 18:10 (5340 days ago) @ George Jelliss

George,-> dhw: "Not believing is one side of agnosticism. The other side is not disbelieving." Exactly, but you need to proportion your degree of belief to the degree of evidence! Just because two different views of a subject can be presented does not mean that they are equally valid, 50:50.-Hence why I tend to explanations of scientific nature. While it is true that the line between "natural" and "supernatural" is a seductively shifting snake, a decision must be made at some point: "If you were to side with something today, which would it be?" We have to fall back on what we *know*, and what we *can* know, or we're simply engaging in speculation and not an act of explanation.

--
\"Why is it, Master, that ascetics fight with ascetics?\"

\"It is, brahmin, because of attachment to views, adherence to views, fixation on views, addiction to views, obsession with views, holding firmly to views that ascetics fight with ascetics.\"

Identity

by dhw, Sunday, September 06, 2009, 12:54 (5339 days ago) @ George Jelliss

Concerning the mind, I wrote among other things: "I want to know what causes the involuntary activities (e.g. a new idea), what enables me to guide the deliberate ones (i.e. what actually endows me with consciousness and will), and what translates the "electrochemical connections" into thought, attitudes etc." I also wrote that the materialist view is "that there has to be a physical explanation, but nobody knows it (yet)."-George replied: "But we do know it to a considerable extent! You need to update your knowledge. We've come a long way since Descartes!"-You kindly recommended two websites which you thought might be helpful (and I do appreciate these references):
http://www.web-us.com/brain/bio_org.html
and http://network.nature.com/groups/bpcc-I found the latter hard to negotiate, but eventually logged onto a lengthy discussion which did not provide any sort of consensus, let alone concrete information. It may be that thanks to my technical incompetence I failed to find the section you wanted me to read. However, the first website was easy to follow, and impressed me for two reasons:-1) The sheer complexity of the connections between the tens of billions of neurons: "these connections have an incredibly complicated structure ... different portions of the brain have different types of connection pattern, while these different sectors of the brain are themselves linked together by further specialized networking." On the occasions when I'm forced to attend religious services (e.g. weddings and funerals), I find my scepticism concerning the benevolent creator figure impossible to suppress. I'm afraid it's the reverse procedure when I read about the above complexities. The author says that "the effective power of the brain is very much larger than current computers", and yet I'm expected to believe that this "incredibly complicated structure" is the result (even over aeons) of processes initiated by sheer chance. -2) We may have come a long way since Descartes, but my initial questions have still not been answered. The author you recommended writes: "The very complexity of these neural networks poses a formidable barrier to understanding. Nobody knows in detail how the individual firings of neurons coupled to their interconnections can lead to all of the features observed ... short and long term memory, complex pattern recognition, logical reasoning, emotion and consciousness. Indeed, it is not known how even the lower level unconscious functions such as those which regulate breathing and heart rate emerge out of the complicated mutual interaction of millions of neurons." This is precisely the point I have been making. We do not know, and it is an act of faith to assume that eventually the materialist explanation of our identity will be confirmed. I note that you would prefer to call your views "scientific", but I do not think that is appropriate. Materialism is a theory, and belief in it requires a high degree of subjectivity. Science purports to be objective, and so "scientific" would endow your belief with a quality it does not have. --I pointed out that agnosticism = not believing and not disbelieving. You replied: "you need to proportion your degree of belief to the degree of evidence! Just because two different views of a subject can be presented does not mean that they are equally valid, 50:50." 
You are right. But you and David both have a scientific background, have proportioned your degrees of belief to the degree of evidence, and have come to diametrically opposite conclusions. I have enormous respect for both of you, have learned and hope to go on learning a great deal from both of you, but ultimately both of you take a leap of faith which I'm not ready to take.-My thanks to David for correcting me on the Abiogenesis thread, and to Matt for the fascinating article on building a model brain. I note in the latter that "the biggest challenge is to understand how electric-magnetic-chemical patterns in the brain convert into our perception of reality." The author might have added consciousness, emotion, memory etc.

Identity

by xeno6696 @, Sonoran Desert, Sunday, September 06, 2009, 15:26 (5339 days ago) @ dhw

My thanks to David for correcting me on the Abiogenesis thread, and to Matt for the fascinating article on building a model brain. I note in the latter that "the biggest challenge is to understand how electric-magnetic-chemical patterns in the brain convert into our perception of reality." The author might have added consciousness, emotion, memory etc.-Well, I mentioned earlier that in forty years (barring some physical constraint, such as what has limited processor speeds to around 3.0Ghz) our ability to create grid computers will result in a machine that is more powerful than the human brain. The article I linked here is the first step towards trying to model the human brain as completely as possible. Once we have enough computing power, this becomes feasible, and breaking that computational barrier is step one. (As a basic rule if you're going to emulate a processor, you want something at least 4x more powerful for it to run smoothly.) -If the suspicion of many a researcher is correct and a consciousness arises from this machine without direct intervention by a human being, it will resolve the origin of consciousness question. This, is a big deal. Do we then grant such sentient machines the same rights as humans?

--
\"Why is it, Master, that ascetics fight with ascetics?\"

\"It is, brahmin, because of attachment to views, adherence to views, fixation on views, addiction to views, obsession with views, holding firmly to views that ascetics fight with ascetics.\"

Identity

by David Turell @, Monday, September 07, 2009, 02:15 (5339 days ago) @ xeno6696

If the suspicion of many a researcher is correct and a consciousness arises from this machine without direct intervention by a human being, it will resolve the origin of consciousness question. This, is a big deal. Do we then grant such sentient machines the same rights as humans?-I'm sorry. I'm with Penrose. If a Pet scan lights up the amygdala, you still don't know all the connections,or exactly how it functions. Pet scans tell us functions of areas. Not how to connect the billions upon billions of neurons thru trillions of synapses. AI will always be AI and there will be no consciousness, no matter how big the computer power

Identity

by xeno6696 @, Sonoran Desert, Monday, September 07, 2009, 06:58 (5338 days ago) @ David Turell

If the suspicion of many a researcher is correct and a consciousness arises from this machine without direct intervention by a human being, it will resolve the origin of consciousness question. This, is a big deal. Do we then grant such sentient machines the same rights as humans?
> 
> I'm sorry. I'm with Penrose. If a Pet scan lights up the amygdala, you still don't know all the connections,or exactly how it functions. Pet scans tell us functions of areas. Not how to connect the billions upon billions of neurons thru trillions of synapses. AI will always be AI and there will be no consciousness, no matter how big the computer power-Faith. And raw, at that. The work here hasn't been done yet. I feel you underestimate human ingenuity! -On this topic I'll accept your claim only when its proven--computers are my raison d'etre, mon ami. I've been reading on neural net computing, and those guys are on to something, and on to something huge. It might end up being perceptual intelligence only, but if we are going to create a consciousness it's going to be resting squarely on that subfield. And if it fails, any hope of sentient AI dies with it--and then you shall be correct. But only then, not before. -Programming for AI is completely different than programming for say, the browser you use. The stuff we use (and I routinely program) is entirely mechanistic and determined. AI programming is exactly the opposite; you have to teach AI to do what it does. We can handle the earliest stages of perceptual intelligence. I tremble at what will become available as we learn how to turn the internet into a neural net.-You'll be noticing over the next 5 years or so, the internet is going to be alot more spooky in how it finds you related information. Web 2.0 technology is building ontological thinking into the internet, a milestone on many levels. Kurzweil's singularity might become a reality. -Optimistic? I'm a technologist after all... only one way for me to be...

--
\"Why is it, Master, that ascetics fight with ascetics?\"

\"It is, brahmin, because of attachment to views, adherence to views, fixation on views, addiction to views, obsession with views, holding firmly to views that ascetics fight with ascetics.\"

Identity

by David Turell @, Monday, September 07, 2009, 14:18 (5338 days ago) @ xeno6696
edited by unknown, Monday, September 07, 2009, 14:23

If the suspicion of many a researcher is correct and a consciousness arises from this machine without direct intervention by a human being, it will resolve the origin of consciousness question. This, is a big deal. Do we then grant such sentient machines the same rights as humans?
> > 
> > I'm sorry. I'm with Penrose. If a Pet scan lights up the amygdala, you still don't know all the connections,or exactly how it functions. Pet scans tell us functions of areas. Not how to connect the billions upon billions of neurons thru trillions of synapses. 
> Faith. And raw, at that. The work here hasn't been done yet. > 
> Optimistic? I'm a technologist after all... only one way for me to be...-And a prediction pops up: making a brain: -Funny he isn't quoted discussing consciousness, but I as interpret it, he is thinking about supercomputing, but implying that how we view reality might be discovered.-http://www.sciencedaily.com/releases/2009/09/090904071908.htm-Apropos of my comment that recognizing a brain area that lights up for a given stimulus doesn't really tell you how it is working 'inside':-http://www.sciencedaily.com/releases/2009/09/090904165447.htm

Identity

by xeno6696 @, Sonoran Desert, Monday, September 07, 2009, 16:20 (5338 days ago) @ David Turell

If the suspicion of many a researcher is correct and a consciousness arises from this machine without direct intervention by a human being, it will resolve the origin of consciousness question. This, is a big deal. Do we then grant such sentient machines the same rights as humans?
> > > 
> > > I'm sorry. I'm with Penrose. If a Pet scan lights up the amygdala, you still don't know all the connections,or exactly how it functions. Pet scans tell us functions of areas. Not how to connect the billions upon billions of neurons thru trillions of synapses. 
> > Faith. And raw, at that. The work here hasn't been done yet. > 
> > Optimistic? I'm a technologist after all... only one way for me to be...
> 
> And a prediction pops up: making a brain: 
> 
> Funny he isn't quoted discussing consciousness, but I as interpret it, he is thinking about supercomputing, but implying that how we view reality might be discovered.
> 
> http://www.sciencedaily.com/releases/2009/09/090904071908.htm
> 
> Apropos of my comment that recognizing a brain area that lights up for a given stimulus doesn't really tell you how it is working 'inside':
> 
> http://www.sciencedaily.com/releases/2009/09/090904165447.htm-His idea ultimately is to build a computer model of a human brain. One of the theories of consciousness thrown out by neural net computing is that consciousness is like the act of bird flight--an emergent property consisting of multiple individual elements, just like the stock market, and just like ecosystems. What causes our consciousness to NOT be flighty, something that is always with us? That's something that we can only understand by modeling the human brain via computing. -Research done on ethics actually has shed some light of approval on neural net theories. It was from a radiolab episode on ethics, but they talk with a (Harvard or Yale ethicist) about some research done on the old-fashioned train scenarios. It seems that when asked to do something morally reprehensible, neurons across the brain act in a discordant and agitated state, as if screaming NO! There are still neurons that seem to light up for "YES!" but the decisions of respondents always side with whichever of the two signals are strongest. This strongly suggests that the mind IS an emergent property of cells. But before you can discount that theory, we need to actually do the work, which is why I pretty hotly contested you last night. The question is not closed.

--
\"Why is it, Master, that ascetics fight with ascetics?\"

\"It is, brahmin, because of attachment to views, adherence to views, fixation on views, addiction to views, obsession with views, holding firmly to views that ascetics fight with ascetics.\"

Identity

by David Turell @, Tuesday, September 08, 2009, 14:44 (5337 days ago) @ David Turell

I'm sorry. I'm with Penrose. If a Pet scan lights up the amygdala, you still don't know all the connections,or exactly how it functions. Pet scans tell us functions of areas. Not how to connect the billions upon billions of neurons thru trillions of synapses. 
> Apropos of my comment that recognizing a brain area that lights up for a given stimulus doesn't really tell you how it is working 'inside':-
Here is an excellent study showing how complicated one neuron is and how connections are modified as we learn. There is a whole system for that emphasis. Still with Penrose.-http://www.sciencedaily.com/releases/2009/09/090907142302.htm

Identity

by xeno6696 @, Sonoran Desert, Wednesday, September 09, 2009, 03:18 (5337 days ago) @ David Turell

I'm sorry. I'm with Penrose. If a Pet scan lights up the amygdala, you still don't know all the connections,or exactly how it functions. Pet scans tell us functions of areas. Not how to connect the billions upon billions of neurons thru trillions of synapses. 
> > Apropos of my comment that recognizing a brain area that lights up for a given stimulus doesn't really tell you how it is working 'inside':
> 
> 
> Here is an excellent study showing how complicated one neuron is and how connections are modified as we learn. There is a whole system for that emphasis. Still with Penrose.
> 
> http://www.sciencedaily.com/releases/2009/09/090907142302.htm-There's another fundamental difference in how you and I think, that masterfully impacts our views; our education. -The problem with a biological approach is that all you can ever do is observe. You can't create. All you can ever do is describe and document complexity. You can accrue data and make correlations, and though you do make models, they are always simple. -Computing (as I mentioned when I first joined) takes an entirely different approach to such a question. It asks us to literally build a model of what it is we're trying to study, because at least in computer science, it can't be said you understand something without being able to build a working model of it. -The complexity seems intractable to you--perhaps for the very reason that the awe you get from the life sciences is in observation of the natural world. A biologist can no more build a tree or an ape. You can study the cells, you can determine the structure of the DNA, you can do a great many things--but because you can't build a *real* model of what you're studying, understanding the... "thing in itself," is an avenue that is naturally closed to you. So you side with Penrose.-The world that I'm in, is one where it is possible to map the complexity of biochemical reactions (such as what that mathematical biology group is doing here at UNO) and believe it or not, this mapping has never been done before. -http://www.pnas.org/content/105/6/1913.full.pdf+html-For the first time--through the "magic" of model-building we're figuring out the complexities of biochemistry. I am not arguing for materialism or immaterialism here; the knowledge that the complexity of life is no longer something that cannot be studied drastically shapes the way I look at these problems. If more information from these studies agrees, cells themselves are a form of AI that can be directly modeled, allowing us to be able to create a great many kinds of models that would never have existed before. (Albeit, no solution to the origins question.)

--
\"Why is it, Master, that ascetics fight with ascetics?\"

\"It is, brahmin, because of attachment to views, adherence to views, fixation on views, addiction to views, obsession with views, holding firmly to views that ascetics fight with ascetics.\"

Identity

by David Turell @, Wednesday, September 09, 2009, 14:28 (5336 days ago) @ xeno6696

Apropos of my comment that recognizing a brain area that lights up for a given stimulus doesn't really tell you how it is working 'inside': -The study shown below identifies an area of the hippocampus that may be a schizophrenia trigger:-http://sciencenow.sciencemag.org/cgi/content/full/2009/908/1-
 
> Computing (as I mentioned when I first joined) takes an entirely different approach to such a question. It asks us to literally build a model of what it is we're trying to study, because at least in computer science, it can't be said you understand something without being able to build a working model of it. 
> 
>> For the first time--through the "magic" of model-building we're figuring out the complexities of biochemistry. -I doubt that we can ever model any area of the brain. Do you do a proxy model or an exact replication of each synaptic connection, with their innate ability to modify. Computers do not live and self-modify.

Identity

by xeno6696 @, Sonoran Desert, Wednesday, September 09, 2009, 18:04 (5336 days ago) @ David Turell

Apropos of my comment that recognizing a brain area that lights up for a given stimulus doesn't really tell you how it is working 'inside': 
> 
> The study shown below identifies an area of the hippocampus that may be a schizophrenia trigger:
> 
> http://sciencenow.sciencemag.org/cgi/content/full/2009/908/1
> 
> 
> 
Awesome, this study. -> > Computing (as I mentioned when I first joined) takes an entirely different approach to such a question. It asks us to literally build a model of what it is we're trying to study, because at least in computer science, it can't be said you understand something without being able to build a working model of it. 
> > 
> >> For the first time--through the "magic" of model-building we're figuring out the complexities of biochemistry. 
> 
> I doubt that we can ever model any area of the brain. Do you do a proxy model or an exact replication of each synaptic connection, with their innate ability to modify. Computers do not live and self-modify.-It might not be a requirement [EDIT] to "live" in the biological sense of the word in order to have consciousness. -Machines most certainly DO self-modify. Anytime you run Java code on your machine the compiler sitting in the virtual machine does this frequently.-http://en.wikipedia.org/wiki/Self-modifying_code-AI is self-modifying in all but its most rudimentary forms. Pattern-recognition allows a machine to solve unknown problems using only the solutions it "knows." When it learns in the first place it is "tabula rasa" and the learning algorithms alter behavior (often using genetic algorithms) and it codes its own solutions. So here, you are incorrect: Computers DO self-modify. You're thinking in the mindset of traditional programming and not AI programming: Completely different skill sets and paradigms. -To my counterparts that study biology, cells are actually quite easily modeled as finite automata. The problem of complexity is that you need a powerful enough network of processor cores (like the conficker network) to adequately model a complex system. The first paper I showed you was the model of a single cell. The processing power required for that one sim is immense. Assuming Moore's law hasn't died yet, in about 20yrs (opposed to the 10 on the original science daily "make a brain from scratch" article, we should have the baseline technology to be able to run our first *true* brain models. -http://en.wikipedia.org/wiki/Finite_automata-Note here, as I seem to run into this: I am not making any claims concerning the origin of consciousness or about the likelihood of a creator. All I'm saying is that the complexity of even things such as the human mind, *may* not be mysteries forever.-EDITED

--
\"Why is it, Master, that ascetics fight with ascetics?\"

\"It is, brahmin, because of attachment to views, adherence to views, fixation on views, addiction to views, obsession with views, holding firmly to views that ascetics fight with ascetics.\"

Identity

by David Turell @, Monday, September 28, 2009, 13:57 (5317 days ago) @ xeno6696

Note here, as I seem to run into this: I am not making any claims concerning the origin of consciousness or about the likelihood of a creator. All I'm saying is that the complexity of even things such as the human mind, *may* not be mysteries forever.-Matt: Hold your nose and look up this site. ID people turn up the most interesting discussions. Read the paper Hunter refers to on the difficulties in understanding the brain's computing functions. It is a pdf file and I don't have the capacity to refer it to this site. Obviously Hunter has a different take than you will.:-))-http://darwins-god.blogspot.com/

Identity

by xeno6696 @, Sonoran Desert, Monday, September 28, 2009, 16:35 (5317 days ago) @ David Turell

Note here, as I seem to run into this: I am not making any claims concerning the origin of consciousness or about the likelihood of a creator. All I'm saying is that the complexity of even things such as the human mind, *may* not be mysteries forever.
> 
> Matt: Hold your nose and look up this site. ID people turn up the most interesting discussions. Read the paper Hunter refers to on the difficulties in understanding the brain's computing functions. It is a pdf file and I don't have the capacity to refer it to this site. Obviously Hunter has a different take than you will.:-))
> 
> http://darwins-god.blogspot.com/-One major criticism:-we believe the problem is not computer power and ability to program parallel machines, but rather our nearly total ignorance about what computations are actually carried out by the brain. Our view is that computers will never equal our best abilities until we can understand the brain's design principles and the mathematical operations employed by neural circuits well enough to build machines that incorporate them.-I contend that we will never "understand the brain's design principles" without a trial-and-error process of literally trying to build a brain. I said it before and I will state it again: Neuroscientists are not going to solve this problem. (I have Adler for backup on that one.) Computer Scientists will. Neuroscience (and biology at large) [EDIT] are descriptive sciences, and you can't do as much with descriptive as constructive. To me, I think he sees the complexity and gives up. -I chuckle a bit at this line too: Religion drives science and it matters. -Yes, religion matters, but the first part of the statement is an empty claim. Few atheists will say that atheism inspires them; less so for agnostics, and we can both agree that there is no greater source of atheists and agnostics than academia. Unless you loosen the term "religion" to mean "awe of nature." But that's clearly diluting the man's point; that's something we all share regardless of theistic predisposition. -On an interesting sidenote, you've used the Einstein quote before about science without religion being lame. Here's something pulled from an old issue of Skeptic:- I have repeatedly said that in my opinion the idea of a personal God is a childlike one. You may call me an agnostic, but I do not share the crusading spirit of the professional atheist whose fervor is mostly due to a painful act of liberation from the fetters of religious indoctrination received in youth. I prefer an attitude of humility corresponding to the weakness of our intellectual understanding of nature and of our own being. - - Albert Einstein, letter to Guy H. Raner Jr., Sept. 28, 1949, quoted by Michael R. Gilmore in Skeptic, Vol. 5, No. 2 -EDITED

--
\"Why is it, Master, that ascetics fight with ascetics?\"

\"It is, brahmin, because of attachment to views, adherence to views, fixation on views, addiction to views, obsession with views, holding firmly to views that ascetics fight with ascetics.\"

Identity

by David Turell @, Monday, September 28, 2009, 19:49 (5317 days ago) @ xeno6696


> > http://darwins-god.blogspot.com/
> 
> One major criticism:
> 
> we believe the problem is not computer power and ability to program parallel machines, but rather our nearly total ignorance about what computations are actually carried out by the brain. Our view is that computers will never equal our best abilities until we can understand the brain's design principles and the mathematical operations employed by neural circuits well enough to build machines that incorporate them.
> 
> I contend that we will never "understand the brain's design principles" without a trial-and-error process of literally trying to build a brain.-It will be trial and error, with 100 billion neurons, self-controlled, with million of miles of axons and dendrites and who knows how many synapses. I admire your optimism but I'm still with Penrose.- 
> I chuckle a bit at this line too: Religion drives science and it matters. -I really intended for you to ignore C. Hunter. I know how nutty he is, but he is bright and does turn up interesting articles for me to review. The moral is: he supplies good stuff to study. He interprets it his way I and I go my way. - 
> On an interesting sidenote, you've used the Einstein quote before about science without religion being lame. Here's something pulled from an old issue of Skeptic:
> 
> I have repeatedly said that in my opinion the idea of a personal God is a childlike one. You may call me an agnostic, but I do not share the crusading spirit of the professional atheist whose fervor is mostly due to a painful act of liberation from the fetters of religious indoctrination received in youth. I prefer an attitude of humility corresponding to the weakness of our intellectual understanding of nature and of our own being. 
> 
> - Albert Einstein, letter to Guy H. Raner Jr., Sept. 28, 1949, quoted by Michael R. Gilmore in Skeptic, Vol. 5, No. 2 -Great comment from Albert, and I think it means the same as my brief quote that I use. He was an agnostic, but interestingly, Israel offerd him the first presidency when that country started. He was a strong supporter of Israel but did not want to be political.

Identity

by xeno6696 @, Sonoran Desert, Tuesday, September 29, 2009, 20:38 (5316 days ago) @ David Turell

I contend that we will never "understand the brain's design principles" without a trial-and-error process of literally trying to build a brain.
> 
> It will be trial and error, with 100 billion neurons, self-controlled, with million of miles of axons and dendrites and who knows how many synapses. I admire your optimism but I'm still with Penrose.
> -Nonsense here, digital logic requires none of those silly dendrites, axons and synapses. Electronic representations are more than sufficient. -> 
> > I chuckle a bit at this line too: Religion drives science and it matters. 
> 
> I really intended for you to ignore C. Hunter. I know how nutty he is, but he is bright and does turn up interesting articles for me to review. The moral is: he supplies good stuff to study. He interprets it his way I and I go my way. 
> 
> 
> > On an interesting sidenote, you've used the Einstein quote before about science without religion being lame. Here's something pulled from an old issue of Skeptic:
> > 
> > I have repeatedly said that in my opinion the idea of a personal God is a childlike one. You may call me an agnostic, but I do not share the crusading spirit of the professional atheist whose fervor is mostly due to a painful act of liberation from the fetters of religious indoctrination received in youth. I prefer an attitude of humility corresponding to the weakness of our intellectual understanding of nature and of our own being. 
> > 
> > - Albert Einstein, letter to Guy H. Raner Jr., Sept. 28, 1949, quoted by Michael R. Gilmore in Skeptic, Vol. 5, No. 2 
> 
> Great comment from Albert, and I think it means the same as my brief quote that I use. He was an agnostic, but interestingly, Israel offerd him the first presidency when that country started. He was a strong supporter of Israel but did not want to be political.-Well, you can take the God out of the Jew, but you'll never take the Jew from the Jew. Just as I wish the best for my Nordic heritage, so did he and his Jewish one. Actually, have you ever taken time to read some of the writings from the Zohar? Some excellent philosophy and theology came out of that monster. (I've only read selections.)

--
\"Why is it, Master, that ascetics fight with ascetics?\"

\"It is, brahmin, because of attachment to views, adherence to views, fixation on views, addiction to views, obsession with views, holding firmly to views that ascetics fight with ascetics.\"

Identity

by David Turell @, Tuesday, September 29, 2009, 22:21 (5316 days ago) @ xeno6696

I contend that we will never "understand the brain's design principles" without a trial-and-error process of literally trying to build a brain.
> > 
> > It will be trial and error, with 100 billion neurons, self-controlled, with million of miles of axons and dendrites and who knows how many synapses. I admire your optimism but I'm still with Penrose.
> > 
> 
> Nonsense here, digital logic requires none of those silly dendrites, axons and synapses. Electronic representations are more than sufficient. -
Yeah, but you will still have to analyze exactly what the brain is doing in each region, how it is modulating each neuron, and how it is growing new connections as it continues to learn from experience. Until all that is understood, you cannot model electronics. And I'm not sure we will ever fully understand all those functions and changes. If we do then you will be right.

Identity

by xeno6696 @, Sonoran Desert, Tuesday, September 29, 2009, 23:25 (5316 days ago) @ David Turell

I contend that we will never "understand the brain's design principles" without a trial-and-error process of literally trying to build a brain.
> > > 
> > > It will be trial and error, with 100 billion neurons, self-controlled, with million of miles of axons and dendrites and who knows how many synapses. I admire your optimism but I'm still with Penrose.
> > > 
> > 
> > Nonsense here, digital logic requires none of those silly dendrites, axons and synapses. Electronic representations are more than sufficient. 
> 
> 
> Yeah, but you will still have to analyze exactly what the brain is doing in each region, how it is modulating each neuron, and how it is growing new connections as it continues to learn from experience. Until all that is understood, you cannot model electronics. And I'm not sure we will ever fully understand all those functions and changes. If we do then you will be right.-By electronic representations, I mean symbolic variables that link neuron a through dendrite v to neuron b, and etc. In terms of building this model, work such as what is going on here at UNO will be foundational, as what they are trying to do is map all of the biochemical reactions into logical a->b->c->... structures. The work is complex, but the work they did on that paper I posted creates a wealth of information that didn't exist before, and the open-source model is only going to improve data collection. -We won't know how the brain does all of those things until we sit down and try to build this model. First we need known biochemical mappings, then we need to integrate them into a large model. Neuroscientists can't do that. -Other research is quite helpful, such as those scientists working to make computers out of biological components:-http://news.bbc.co.uk/2/hi/science/nature/358822.stm-By approaching the problem from both ends (in the practical manner of building computers and building models) we'll understand more about the brain than biochemistry alone can possibly provide. -In terms of analysis, techniques to analyze cognition are already in place by my AI friends; AI has been a much greater driver of cognitive research than any other method the last 30 years. We won't improve those models by studying petri dishes!!

--
\"Why is it, Master, that ascetics fight with ascetics?\"

\"It is, brahmin, because of attachment to views, adherence to views, fixation on views, addiction to views, obsession with views, holding firmly to views that ascetics fight with ascetics.\"

Identity

by David Turell @, Wednesday, September 30, 2009, 05:14 (5316 days ago) @ xeno6696


> By electronic representations, I mean symbolic variables that link neuron a through dendrite v to neuron b, and etc. In terms of building this model, work such as what is going on here at UNO will be foundational, as what they are trying to do is map all of the biochemical reactions into logical a->b->c->... structures. > 
> We won't know how the brain does all of those things until we sit down and try to build this model. First we need known biochemical mappings, then we need to integrate them into a large model. Neuroscientists can't do that. 
> 
> Other research is quite helpful, such as those scientists working to make computers out of biological components:
> 
> http://news.bbc.co.uk/2/hi/science/nature/358822.stm
> 
> By approaching the problem from both ends (in the practical manner of building computers and building models) we'll understand more about the brain than biochemistry alone can possibly provide. 
> 
> In terms of analysis, techniques to analyze cognition are already in place by my AI friends; -The link is very interesting, but basically I don't have the background to follow what you are describing in the computer and model building.

Identity

by xeno6696 @, Sonoran Desert, Wednesday, September 30, 2009, 05:58 (5315 days ago) @ David Turell

David,-I'll do my best to explain this... it'll actually help me a bit as I'm moving into some more "advanced data structures" in my program. -In computer science, the simplest type of data structures are arrays. In short, they are a list identified by an index. Item 0 through x. The only way to access the data is to pull something in by its index. Most languages won't allow you to break that rule... though this year I've been introduced to one that lets you break a ton of rules. (Beyond scope, BAD matt...) -The next level up are linked lists. The difference between linked lists and arrays, is that there is no need for an index. As each data object is created it is linked to the previous one in the chain. The benefit here is that you don't need to have an index... the drawback is that you have to crawl through each 'node' in order find out where your desired data object is.-The node is the central concept in neural-net programming--and in more 'generic' types of programming. A higher level of data structure complexity is the graph; where a linked-list can be viewed as a line of cups, a graph is better represented by a spider's web. (If you Wikipedia "graph theory" it'll give you enough background to get the flavor.) Each node can be connected (by memory reference) to any number of other nodes. In terms of modeling something biologically, this has tremendous advantages as we can model a neuron by creating a node, and simply storing all of its individual references.-Neural net programming involves creating a series of nodes and limiting their individual abilities while allowing emergent behaviors to play. While a symphony is more than its pieces, so are neural nets. Emergent properties emerge, and it turns out that neural nets tend to make better decisions than we can "forcefully" program. -The basics are actually explained pretty well here: http://en.wikipedia.org/wiki/Artificial_neural_network
It loses some coherency in the areas where they start bringing up formulas. Not that I can't piece them together, but the writing style is... lackluster. In about a year I should be able to say more on the subject beyond a 30k ft overview. -Modeling a human brain would essentially mean that we have several billion blocks of memory connected in various paths to each other. The biochemistry COULD be modeled, but to me that would be unnecessary complexity at this point. We need a 'best case' working model before we can start worrying about the finer chemical complexities... that we don't understand yet. We need both UNO's and Yale's systems to collect a ton of data before we can really do that part of it justice. -The basic ideas are there, the computer languages we have available are in my opinion, sophisticated enough to allow us to give this a real try. While I mute my optimism, there can be no better way to understand something except from trying to build it. Of course, this is the humble opinion of an engineer, so take that as my own brand of perspectivism.

--
\"Why is it, Master, that ascetics fight with ascetics?\"

\"It is, brahmin, because of attachment to views, adherence to views, fixation on views, addiction to views, obsession with views, holding firmly to views that ascetics fight with ascetics.\"

Identity

by David Turell @, Wednesday, September 30, 2009, 16:54 (5315 days ago) @ xeno6696

Modeling a human brain would essentially mean that we have several billion blocks of memory connected in various paths to each other. The biochemistry COULD be modeled, but to me that would be unnecessary complexity at this point. We need a 'best case' working model before we can start worrying about the finer chemical complexities... that we don't understand yet. We need both UNO's and Yale's systems to collect a ton of data before we can really do that part of it justice. 
> 
> The basic ideas are there, the computer languages we have available are in my opinion, sophisticated enough to allow us to give this a real try. While I mute my optimism, there can be no better way to understand something except from trying to build it. Of course, this is the humble opinion of an engineer, so take that as my own brand of perspectivism.-Thanks for the explanation. The biochemistry simply arranges for the ions to travel, and modifications of the neuron's output. Mimic that and the biochemistry is not important.

Identity

by dhw, Monday, September 07, 2009, 17:30 (5338 days ago) @ xeno6696

Matt has informed us about the project to build a machine more powerful than the human brain. "If the suspicion of many a researcher is correct and a consciousness arises from this machine without direct intervention by a human being, it will resolve the origin of consciousness question."-David responded: "AI will always be AI and there will be no consciousness, no matter how big the computer power." Matt's response to this is: "Faith. And raw at that. The work here hasn't been done yet. I feel you underestimate human ingenuity! On this topic I'll accept your claim only when it's proven."-Can it be proven that consciousness is NOT purely material? Can it be proven that life could NOT be the product of chance? If one experiment fails, scientists will move onto another. Faith is part of both sets of beliefs, and I am full of admiration for the scientists who pursue these dreams. Man's quest for knowledge is one of the qualities that make me proud to be human, and it will be fascinating to see what the model brain can and can't achieve.-However, you say, Matt, that if consciousness arises, it will resolve the origin of consciousness question. This needs closer scrutiny. First of all, what you are saying amounts to this: if scientists can prove that consciousness is created solely by material means, it will prove that consciousness is created solely by material means. I too find the project exciting, but the hypothesis itself doesn't exactly have me jumping up and down. Secondly, the word 'origin' is misleading. If intelligent humans really do produce a sentient, conscious machine, the achievement will be monumentally, sensationally brilliant, and awards will tumble down on them like confetti. But the feat will not resolve the origin of consciousness question. It will show us that consciousness can be produced by materials, but that it takes super intelligence to put those materials together. -What such a machine would achieve for me personally, though, is proof that we do not need a non-material explanation of identity through concepts like soul or spirit. The mechanism (in this case, I think George will accept the word) will have produced its own will, emotions etc. This would definitely require a rethink, also on the part of theists. But Matt, if I may quote your response to David, "on this topic, I'll accept your claim only when it's proven". Until then, the materialist interpretation of mind and identity is "Faith. And raw, at that." -P.S. You wrote: "Computers are my raison d'être, mon ami." Je vous conseille, mon ami, de ne pas le dire à votre femme.

Identity

by xeno6696 @, Sonoran Desert, Monday, September 07, 2009, 20:01 (5338 days ago) @ dhw

dhw, fascinating as usual! 
> Can it be proven that consciousness is NOT purely material? Can it be proven that life could NOT be the product of chance? If one experiment fails, scientists will move onto another. Faith is part of both sets of beliefs, and I am full of admiration for the scientists who pursue these dreams. Man's quest for knowledge is one of the qualities that make me proud to be human, and it will be fascinating to see what the model brain can and can't achieve.
> -Consciousness itself *is* immaterial. There is no way to refute that. Its origins however... who knows at this point? We know that the brain definitely is its seat--to be fair--most of the time. You are correct that if one experiment fails, scientists move on to another, but neural net computing is the closest artifact man has created to the basic building blocks of the mind. If a consciousness is created in this (artifact is the "proper" term), it won't absolutely solve the origins question, but it would solve the "difference of man" question most assuredly. If this long-term experiment fails, there is no realistic means left to *study* consciousness. If we can't recreate it we can't really *know* anything about it. -> However, you say, Matt, that if consciousness arises, it will resolve the origin of consciousness question. This needs closer scrutiny. First of all, what you are saying amounts to this: if scientists can prove that consciousness is created solely by material means, it will prove that consciousness is created solely by material means. I too find the project exciting, but the hypothesis itself doesn't exactly have me jumping up and down. Secondly, the word 'origin' is misleading. If intelligent humans really do produce a sentient, conscious machine, the achievement will be monumentally, sensationally brilliant, and awards will tumble down on them like confetti. But the feat will not resolve the origin of consciousness question. It will show us that consciousness can be produced by materials, but that it takes super intelligence to put those materials together. 
> -Agreed. However it will also show that it is true that only a particular threshold needs to be reached in brain complexity, and that the difference between us and everything else (in terms of consciousness) is only in degree. It means that consciousness sould be relatively common in the universe. Does it resolve the question that brings us to the site? No. It gives me more reason to doubt a creator, because I think the strongest arguments for a creator lie in the concepts of consciousness. It won't give us an answer to how *our* consciousness arose, but it will give us insights that we don't have.-> What such a machine would achieve for me personally, though, is proof that we do not need a non-material explanation of identity through concepts like soul or spirit. The mechanism (in this case, I think George will accept the word) will have produced its own will, emotions etc. This would definitely require a rethink, also on the part of theists. But Matt, if I may quote your response to David, "on this topic, I'll accept your claim only when it's proven". Until then, the materialist interpretation of mind and identity is "Faith. And raw, at that." 
> -Note: I don't claim they'll be successful. (That would be raw faith.) I was only analyzing the case if it succeeded or failed. I don't even think they will be successful. But the possibilities of success open the imagination to new vistas, as such it is exciting. David to me seems to be prejudging the work. I can't do that. No one has actually *tried* to do it. Let them try, then judge. -
> P.S. You wrote: "Computers are my raison d'être, mon ami." Je vous conseille, mon ami, de ne pas le dire à votre femme.-Ahh... my french is really bad, but I think you said "don't say that to my wife!" If I got that right I laughed my butt off because I thought that after I typed it. I didn't remember how to say "one of my." I think I might pick French back up after I graduate. In the meantime, I'm learning Chinese. (My best friend is bringing a Chinese wife home next summer and my wife and I want to ease the transition as much as possible.)

--
\"Why is it, Master, that ascetics fight with ascetics?\"

\"It is, brahmin, because of attachment to views, adherence to views, fixation on views, addiction to views, obsession with views, holding firmly to views that ascetics fight with ascetics.\"

Identity

by David Turell @, Monday, September 07, 2009, 21:53 (5338 days ago) @ xeno6696

I don't even think they will be successful. But the possibilities of success open the imagination to new vistas, as such it is exciting. David to me seems to be prejudging the work. I can't do that. No one has actually *tried* to do it. Let them try, then judge. -
I'm really not prejudging any more than you have. You seem to imply that I am saying 'why try'. No, I want the research done. That is the only way we will find out. But I feel my prediction they will fail is most accurate.

Identity

by xeno6696 @, Sonoran Desert, Monday, September 07, 2009, 22:32 (5338 days ago) @ David Turell

I don't even think they will be successful. But the possibilities of success open the imagination to new vistas, as such it is exciting. David to me seems to be prejudging the work. I can't do that. No one has actually *tried* to do it. Let them try, then judge. 
> 
> 
> I'm really not prejudging any more than you have. You seem to imply that I am saying 'why try'. No, I want the research done. That is the only way we will find out. But I feel my prediction they will fail is most accurate.-Well, in the long run--on this question--we're in the same boat. At the same time, the mathematics behind neural net computing are quite elegant, and in learning about some of the models and basic algorithms I see tremendous promise. If this doesn't do it, nothing will.-Just in: Statisticians are being predicted to be the next big push in industrial computing. OT but if anyone has kids/grandkids in college and they're wondering what to do:-http://www.nytimes.com/2009/08/06/technology/06stats.html

--
\"Why is it, Master, that ascetics fight with ascetics?\"

\"It is, brahmin, because of attachment to views, adherence to views, fixation on views, addiction to views, obsession with views, holding firmly to views that ascetics fight with ascetics.\"

Identity

by David Turell @, Tuesday, September 08, 2009, 05:24 (5338 days ago) @ xeno6696


> Well, in the long run--on this question--we're in the same boat. At the same time, the mathematics behind neural net computing are quite elegant, and in learning about some of the models and basic algorithms I see tremendous promise. -Our differences are age related. You have the next 50-60 years or so to learn with science whatever will turn up. You can reach conclusions later. I don't have that time, so I have to find conclusions that satisfy me before I move on and either find out at that point or not at all. The only problem is if this website is still active, I won't be able to communicate back, but I'll try.-One of my regrets is not ever learning calculus. I would love to understand what 'beautiful math' is. I know if it is elegant enough, it often predictively leads to the final answer.

Identity

by xeno6696 @, Sonoran Desert, Tuesday, September 08, 2009, 14:17 (5337 days ago) @ David Turell
edited by unknown, Tuesday, September 08, 2009, 14:24


> > Well, in the long run--on this question--we're in the same boat. At the same time, the mathematics behind neural net computing are quite elegant, and in learning about some of the models and basic algorithms I see tremendous promise. 
> 
> Our differences are age related. You have the next 50-60 years or so to learn with science whatever will turn up. You can reach conclusions later. I don't have that time, so I have to find conclusions that satisfy me before I move on and either find out at that point or not at all. The only problem is if this website is still active, I won't be able to communicate back, but I'll try.
> 
> One of my regrets is not ever learning calculus. I would love to understand what 'beautiful math' is. I know if it is elegant enough, it often predictively leads to the final answer.-"Mathematical elegance" is typically a moving target, though to my experience it has meant "beautifully simple," such as Euclid's proof of the irrationality of the square root of 2, or any number of proofs from the more "forgotten" mathematicians Eudoxus or Euler. In computer science, it is more to do with a mixture of that combined with clever logic, and coded with as few lines as possible. Computer guys are notoriously lazy, so inventing a way to do the same work with fewer lines is a pretty strong drive for most of us.-[EDIT]-As for the rest of the post, I would say the difference is more in personal preference. Mathematics has instilled in me an obsession for telling the absolute truth. If a question is unresolved, I feel I'm lying by taking the step towards "belief," and am much more happy leaving it in place exactly as it is in its ugly unanswered state. As I grow older I only notice that drive becoming more and more strong. This has the result of making me seem indecisive, but in the end most people seem to appreciate my sometimes brutal honesty.

--
\"Why is it, Master, that ascetics fight with ascetics?\"

\"It is, brahmin, because of attachment to views, adherence to views, fixation on views, addiction to views, obsession with views, holding firmly to views that ascetics fight with ascetics.\"

Identity

by dhw, Wednesday, September 09, 2009, 14:08 (5336 days ago) @ xeno6696

Matt believes that a man-made machine with consciousness will show that "the difference between us and everything else (in terms of consciousness) is only in degree. It means that consciousness should be relatively common in the universe."-I can't follow the connection with the rest of the universe. One has to bear in mind that our scientists already have a model to copy. If there is no creator or universal intelligence, Nature would either have to keep starting spontaneously from scratch on each planet, or life would have to travel spontaneously between planets and then evolve as it has done on Earth. How would the machine support either eventuality?-This provides an interesting link to your next observation: "It gives me more reason to doubt a creator, because I think the strongest arguments for a creator lie in the concepts of consciousness." I don't see how you can separate consciousness from life. Your Swiss professor says: "When we first switched it on it already started to display some interesting emergent properties." All he had to do was press the button, and the Schweizer Stromversorgungsgesellschaft (or whatever it's called ... I love those compound nouns) did the rest. We not only get switched on and display our emergent properties, but we also manage to reproduce, and to indulge in countless other amazing conscious and unconscious activities. I would therefore argue that the greater the complexity, the stronger the argument against chance, and consciousness is only one part of the equation. -To sum up, I'd say that a man-made, self-aware machine will only prove that consciousness does not require more than the materials of the brain, but that won't tell us anything about (a) there being life or consciousness elsewhere in the universe, (b) how the materials got put together in the first place, and hence (c) the odds for and against there being a creator. But that doesn't make the experiment any the less exciting! -In your latest post to David you comment: "It can't be said you understand something without being able to build a working model of it." (I hope you don't mean my doctor should be able to build a working model of my body!) This echoes the comment in an article recommended by George and quoted by me under "Abiogenesis" on 5 September at 11.50: "The dream of physicists is to create elementary life," Libchaber says. "Then we would know that we understand something." As you so rightly pointed out in an earlier post, the work hasn't been done yet, which means that so far we are not in a position to understand, let alone explain, consciousness or the origin of life. Until we are, I will join you in "not arguing for materialism or immaterialism". -Quick French lesson: "une de mes raisons d'être" would have kept you out of trouble with your wife. Meanwhile, good luck with your Chinese. You'll be pleased to hear that I can't go beyond the parameters of the take-away.

Identity

by xeno6696 @, Sonoran Desert, Wednesday, September 09, 2009, 14:34 (5336 days ago) @ dhw
edited by unknown, Wednesday, September 09, 2009, 14:42

Matt believes that a man-made machine with consciousness will show that "the difference between us and everything else (in terms of consciousness) is only in degree. It means that consciousness should be relatively common in the universe."
> 
> I can't follow the connection with the rest of the universe. One has to bear in mind that our scientists already have a model to copy. If there is no creator or universal intelligence, Nature would either have to keep starting spontaneously from scratch on each planet, or life would have to travel spontaneously between planets and then evolve as it has done on Earth. How would the machine support either eventuality?
> 
> This provides an interesting link to your next observation: "It gives me more reason to doubt a creator, because I think the strongest arguments for a creator lie in the concepts of consciousness." I don't see how you can separate consciousness from life. Your Swiss professor says: "When we first switched it on it already started to display some interesting emergent properties." All he had to do was press the button, and the Schweizer Stromversorgungsgesellschaft (or whatever it's called ... I love those compound nouns) did the rest. We not only get switched on and display our emergent properties, but we also manage to reproduce, and to indulge in countless other amazing conscious and unconscious activities. I would therefore argue that the greater the complexity, the stronger the argument against chance, and consciousness is only one part of the equation. 
> -I think it could be safely said that to me, the strongest arguments for a creator lie in those problems that really are intractable; consciousness for one. If we can create consciousness from inert matter (a computer), it does a couple things. First it lowers the bar on what we consider life. Maybe Gaia really does exist? And it therefore means that it is less likely that we humans are a one-off anomaly in the history of space and time. (Because of said lowered bar.) -
> To sum up, I'd say that a man-made, self-aware machine will only prove that consciousness does not require more than the materials of the brain, but that won't tell us anything about (a) there being life or consciousness elsewhere in the universe, (b) how the materials got put together in the first place, and hence (c) the odds for and against there being a creator. But that doesn't make the experiment any the less exciting! 
> -Answer to (a): if we can create consciousness from inert matter, it serves as positive proof that consciousness can be made from other materials. The lower bar means we *should* be more likely to find it.-(b): I've got nothing here. I think I admitted as much, but if I didn't I do here. We're no closer to solving the origins problem.-(c): Agreed. I wasn't trying to make that argument, but I can see where it can be inferred. It'll prove that intelligent beings can make intelligent beings. Perhaps in this case it serves as a better argument for David. -> In your latest post to David you comment: "It can't be said you understand something without being able to build a working model of it." (I hope you don't mean my doctor should be able to build a working model of my body!) This echoes the comment in an article recommended by George and quoted by me under "Abiogenesis" on 5 September at 11.50: "The dream of physicists is to create elementary life," Libchaber says. "Then we would know that we understand something." As you so rightly pointed out in an earlier post, the work hasn't been done yet, which means that so far we are not in a position to understand, let alone explain, consciousness or the origin of life. Until we are, I will join you in "not arguing for materialism or immaterialism". 
> -David has flatly stated on one occasion that what he did as a doctor was make it easier for the body to do its work. We're just now becoming capable of harnessing the power of stem cells--the "force" needed for the body to do its work, and only in the past 2 years have we discovered that the human appendix isn't useless. And in a recent post to David I also mention that we're just now becoming capable of studying the massive networks of biochemical reactions in a single cell. I rest my case... the human body still has enough secrets that we can't say that we *know* it. -> Quick French lesson: "une de mes raisons d'être" would have kept you out of trouble with your wife. Meanwhile, good luck with your Chinese. You'll be pleased to hear that I can't go beyond the parameters of the take-away.-As much as I love my subject, I've been finding school getting in the way of really getting to know a variety of people from different cultures. I live in the midwest, where in my observation we are very culturally homogeneous. Omaha itself is such a segregated city that we're studied by sociologists far and wide. I've made some resolutions about this but I *must* finish school first. Learning Mandarin is step 1. Then probably Spanish. I've had a long-time goal of being trilingual, but life is too short not to keep jumping in.

--
\"Why is it, Master, that ascetics fight with ascetics?\"

\"It is, brahmin, because of attachment to views, adherence to views, fixation on views, addiction to views, obsession with views, holding firmly to views that ascetics fight with ascetics.\"

Identity

by David Turell @, Tuesday, September 15, 2009, 14:38 (5330 days ago) @ xeno6696

Matt believes that a man-made machine with consciousness will show that "the difference between us and everything else (in terms of consciousness) is only in degree. It means that consciousness should be relatively common in the universe."-
My position has been as a doubter. With the brain's 100 billion cells and innumerable connecting dendrites, axons, and synapses now it is found that individual neurons have two levels of decision-making.-http://www.sciencedaily.com/releases/2009/09/090914172650.htm

Identity

by xeno6696 @, Sonoran Desert, Tuesday, September 15, 2009, 21:46 (5330 days ago) @ David Turell

Matt believes that a man-made machine with consciousness will show that "the difference between us and everything else (in terms of consciousness) is only in degree. It means that consciousness should be relatively common in the universe."
> 
> 
> My position has been as a doubter. With the brain's 100 billion cells and innumerable connecting dendrites, axons, and synapses now it is found that individual neurons have two levels of decision-making.
> 
> http://www.sciencedaily.com/releases/2009/09/090914172650.htm-Again, no problems with doubting.-Important criticisms on this article: It isn't offered as a definitive finding, only another potential model. -From the computational side of things, it wouldn't add much complexity to the programming; only the power of the machines we'd need to have to model it. Axons would be a special subclass of cell that would run a quorum algorithm using fuzzy logic. (Quorum as in "quorum sensing" in bacteria and fuzzy logic as in "makes decisions based on statistical inference.") -From what I've been reading on neural net programming, this "electoral college" phenomenon might be an emergent property and not necessary to specifically model for. It will be interesting to hear their take on this.

--
\"Why is it, Master, that ascetics fight with ascetics?\"

\"It is, brahmin, because of attachment to views, adherence to views, fixation on views, addiction to views, obsession with views, holding firmly to views that ascetics fight with ascetics.\"

Identity

by xeno6696 @, Sonoran Desert, Friday, August 28, 2009, 16:29 (5348 days ago) @ dhw
edited by unknown, Friday, August 28, 2009, 16:39

dhw,
> I wrote that I was "absolutely not prepared to dismiss people's personal experiences as possible evidence". Matt replied: "A person's experiences are inviolable. Your experiences are not open to study by me, because there is no way to recreate the event except in words." And also: "The act of recounting an event from memory is itself always worth casting doubt on. [...] This means that the level of subjectivity is simply too high for my analytical taste."
> 
> In your response to David (26 August at 01.43) you also wrote "To me, any acceptable explanation must be studyable."
> 
> All things are "studyable", but I suspect that what you mean by that is scientifically testable. It comes down to the usual question of whether you do or don't believe that science holds all the answers ... and that in turn comes down to whether you are or are not a materialist. (I'm aware that you are not, but when asked to lean, you tend to go in that direction.) "Subjectivity" is a dirty word in scientific contexts, but I would say that subjective experiences provide us with most of the realities that make life worth living. I hope you didn't insist on a scientific study of the cerebral cortices of your wife and yourself before you told her you loved her. 
> - I think it would be best here to give you the terms I use; subjective experiences are something you can reflect on, but to study something you need to have something "not just in your head" in order to do it. You can write your subjective experiences down; but what you have is only a metaphor of the actual event. So maybe yes, I do mean "scientifically." I do not mean to move subjective experiences to the side--I'm fully aware that it is exactly those things that give our lives meaning. However I feel I would be lying to say that I can *study* my brain. I can reflect on what's in it. I can examine it and try to make sense of it, but study just isn't the right word for that. Rest assured, that I wouldn't have married my wife if I wasn't certain that it was requited love; so while not studying her cortices, I was certainly mining them for information, heh. - > Subjective does not mean unreal or untrue ... and I know you're not saying it does ... but I recall in one of your earlier posts that you dismissed all "paranormal" experiences as fake. (Forgive me if that was not the precise word ... I'm too lazy to go hunting for it ... but it was certainly something like that.). You're right to avoid personal conflict in individual cases, and in general it seems to me quite fair to suspend judgement. But without knowing the facts, it's unfair to pass judgement, let alone impose prejudgement. I wrote that I was absolutely not prepared to dismiss people's personal experiences as possible evidence. Your reply suggests, in the nicest possible way, that you are.
> - A better way to put it would be that I am instinctively skeptical, but that I'm aware that being that way can cause undue stress on friends, family, etc. I'm interested in learning these things from people because they make each of us who we are, but I dare not in any event treat them as something I can study. We are equipped to examine our own subjective experiences, but the moment my buddy "Fred" takes his demon story to me for some kind of validation, he's making it an objective event, and it patently cannot be an objective event, and my skepticism will prevent me from treating it as an objective event. I don't pass judgment, but recognize that no matter how important, I have no right or claim against Fred's demons because unless I was there, I cannot say if it was real or not. Maybe that helps you place my thought a little better? - > I would add, though, that we're ALL subject to personal experiences we don't understand and can't analyse objectively, but because we're so used to them, we're largely unaware of them. That's the subject I'm trying to grapple with under this thread, but it's very difficult to articulate. I shall have another go at it in due course. Your inability and my own to answer the questions I posed on 25 August at 07.55 lie at the heart of it. - Humans have their own "three-body problem" when it comes to ideas. Someone picks up a doctrine and then everything in the world moves according to it. Someone else picks up one of the "orbiting" doctrines and then there's a new interpretation of the same movement. We need to have some kind of common ground to connect experiences to make them "objective." - [EDIT]
When Fred takes his demon story to me, all I can do is provide what I think based on my perspective. However, it is two things, firstly, it is my examination based on his words, which may or may not be wholly accurate. Secondly, his perspective shapes his retelling of the events, and further removes me from the actual event itself. There is far too much subjectivity here to be able to *study* it.

--
\"Why is it, Master, that ascetics fight with ascetics?\"

\"It is, brahmin, because of attachment to views, adherence to views, fixation on views, addiction to views, obsession with views, holding firmly to views that ascetics fight with ascetics.\"

Identity

by David Turell @, Saturday, August 29, 2009, 01:35 (5348 days ago) @ xeno6696

[EDIT]
> When Fred takes his demon story to me, all I can do is provide what I think based on my perspective. However, it is two things, firstly, it is my examination based on his words, which may or may not be wholly accurate. Secondly, his perspective shapes his retelling of the events, and further removes me from the actual event itself. There is far too much subjectivity here to be able to *study* it. - I think you are perfectly right about Fred. All he is telling you is self-contained material. But in many near to death (NDE) episodes, events occur which the comatose patient describes accurately as far as third party witnesses are concerned. The same is true in out of the body (OOB)experiences with third person corroboration of the events. A study is proceeding now with hospices to collect a large number of these. I've mentioned this before, but a collation of these events may yield some interesting material that can be considered 'scientific results'. If an event is not corroborated, then, of course, that event remains subjective.

Identity

by xeno6696 @, Sonoran Desert, Saturday, August 29, 2009, 04:12 (5348 days ago) @ David Turell

[EDIT]
> > When Fred takes his demon story to me, all I can do is provide what I think based on my perspective. However, it is two things, firstly, it is my examination based on his words, which may or may not be wholly accurate. Secondly, his perspective shapes his retelling of the events, and further removes me from the actual event itself. There is far too much subjectivity here to be able to *study* it.
> 
> I think you are perfectly right about Fred. All he is telling you is self-contained material. But in many near to death (NDE) episodes, events occur which the comatose patient describes accurately as far as third party witnesses are concerned. The same is true in out of the body (OOB)experiences with third person corroboration of the events. A study is proceeding now with hospices to collect a large number of these. I've mentioned this before, but a collation of these events may yield some interesting material that can be considered 'scientific results'. If an event is not corroborated, then, of course, that event remains subjective. - There's an absolutely *awesome* scientific theory for NDE's... I can tell that you haven't checked my radiolab podcasts, go to radiolab.org, find the show "Where am I" from the list, and have a blast... research right from the mouth of an USAF doc that's spent something like 10-15 yrs studying the phenomenon.

--
\"Why is it, Master, that ascetics fight with ascetics?\"

\"It is, brahmin, because of attachment to views, adherence to views, fixation on views, addiction to views, obsession with views, holding firmly to views that ascetics fight with ascetics.\"

Identity

by dhw, Tuesday, September 01, 2009, 20:23 (5344 days ago) @ xeno6696

Matt recommended "an absolutely awesome theory for NDE's" from the show "Where am I", on radiolab.org.-Thanks to my technical incompetence, I wasted about an hour trying to find the right show and the right part of the show! A doctor gets pilots to sit in a centrifuge which rotates at colossal speeds. A lot of them then have weird, dreamlike visions unconnected with reality (don't drugs and alcohol have a similar effect?), like fishing or being in a supermarket, and some of them experience OBEs afterwards, being above or behind themselves, and seeing tunnels with a white light (also experienced in NDEs). -The doctor does not draw any conclusions, except that "the brain is confused at having lost the body". This, however, sheds no light on the aspect of NDEs that I find particularly intriguing, which is that some patients obtain information not known at the time but later confirmed by independent witnesses. My main question to you though, Matt, is: what exactly is the "awesome theory"?

Identity

by xeno6696 @, Sonoran Desert, Tuesday, September 01, 2009, 22:55 (5344 days ago) @ dhw

Matt recommended "an absolutely awesome theory for NDE's" from the show "Where am I", on radiolab.org.
> 
> Thanks to my technical incompetence, I wasted about an hour trying to find the right show and the right part of the show! A doctor gets pilots to sit in a centrifuge which rotates at colossal speeds. A lot of them then have weird, dreamlike visions unconnected with reality (don't drugs and alcohol have a similar effect?), like fishing or being in a supermarket, and some of them experience OBEs afterwards, being above or behind themselves, and seeing tunnels with a white light (also experienced in NDEs). 
> 
> The doctor does not draw any conclusions, except that "the brain is confused at having lost the body". This, however, sheds no light on the aspect of NDEs that I find particularly intriguing, which is that some patients obtain information not known at the time but later confirmed by independent witnesses. My main question to you though, Matt, is: what exactly is the "awesome theory"?-Well crap, I must be mixing 2 shows together. Either that or I inferred it and thought it was his idea... no clue. I can't tell you which show from there, but the theory I heard was that our brain is operating based on its own model at all instances. So when those pilots were walking "behind themselves" this is because their brain's model isn't at a 1:1 correspondence to the world, they're still receiving sensory data but the brain is processing it incorrectly. In an OBE/NDE the theory was carried forward, suggesting that the brain was still receiving inputs and recording information that is then recalled upon awaking. This would be supported in my view by the idea that I've never heard of a person recalling events that weren't happening around them, or from someone who was "braindead" for more than a few minutes. -I could also suggest (my idea) that in the low blood-content of the brain in these instances breaks the barrier that allows our subconscious brain to take over the model, allowing you to have a "waking dream," that could easily have you floating above the yourself. This isn't drastically different than the USAF doc "walking behind himself" at all.

--
\"Why is it, Master, that ascetics fight with ascetics?\"

\"It is, brahmin, because of attachment to views, adherence to views, fixation on views, addiction to views, obsession with views, holding firmly to views that ascetics fight with ascetics.\"

Identity

by David Turell @, Wednesday, September 02, 2009, 01:33 (5344 days ago) @ xeno6696

I've never heard of a person recalling events that weren't happening around them, or from someone who was "braindead" for more than a few minutes. 
> 
> I could also suggest (my idea) that in the low blood-content of the brain in these instances breaks the barrier that allows our subconscious brain to take over the model, allowing you to have a "waking dream," that could easily have you floating above the yourself. This isn't drastically different than the USAF doc "walking behind himself" at all.-There are many documented cases with no EKG activity, no EEG brain waves, and the person can give testamony as to what was happening around them, after resuscitation. Pim van Lommel's article in Lancet, I've referred to in the past is a great example. Vol: 358, Dec.15th, 2001.

Identity

by xeno6696 @, Sonoran Desert, Wednesday, September 02, 2009, 02:07 (5344 days ago) @ David Turell

I've never heard of a person recalling events that weren't happening around them, or from someone who was "braindead" for more than a few minutes. 
> > 
> > I could also suggest (my idea) that in the low blood-content of the brain in these instances breaks the barrier that allows our subconscious brain to take over the model, allowing you to have a "waking dream," that could easily have you floating above the yourself. This isn't drastically different than the USAF doc "walking behind himself" at all.
> 
> There are many documented cases with no EKG activity, no EEG brain waves, and the person can give testamony as to what was happening around them, after resuscitation. Pim van Lommel's article in Lancet, I've referred to in the past is a great example. Vol: 358, Dec.15th, 2001.-But no EEG waves doesn't mean that the individual cells aren't doing some work, only that we have no reason to believe the mind is active. The mind doesn't have to be active for this idea to work. If information is being absorbed (as has happened in some anecdotal coma awakenings), all that needs to happen is for the person to come back awake and the combination of collected inputs gets synthesized into an experience--just like when remembering a dream.-I say this because its not like the ears aren't sending sensory input into the brain. It seems that it would make sense if some of that information is still transmitted to some brain cells even if the mind is performing any processing on it. Realistically all I typically need to imagine something is sound, in fact a recent study was just published showing that listening to music with eyes closed heightens emotional response. I can hear words and build a picture around them.-If I remember right the hypothalamus plays a large role in translating memory and is part of the limbic system... I don't think EEG's record activity at that low a level.-At this point I'm more or less letting my mind flow freely here and feel free to stuff whatever's totally off base.

--
\"Why is it, Master, that ascetics fight with ascetics?\"

\"It is, brahmin, because of attachment to views, adherence to views, fixation on views, addiction to views, obsession with views, holding firmly to views that ascetics fight with ascetics.\"

Identity: Skepdic on NDE:

by xeno6696 @, Sonoran Desert, Wednesday, September 02, 2009, 19:41 (5343 days ago) @ David Turell

Searching for some info that might support my previous idea I came across:-http://www.skepdic.com/nde.html-At first it appears an ad-hominem against one particular doctor but from Moody on it presents a balanced view including some articles posted in Lancet. -One thing spoken that took the words out of my mouth: -"Reports of Christians meeting Muhammad or Muslims meeting Christ or Jews meeting Guru Nanak, if they exist, have not been publicized."-Why is it that NDE's are always... "personalized," like dreams?-[EDIT] And apparently Ketamine can be used to completely recreate the experience!

--
\"Why is it, Master, that ascetics fight with ascetics?\"

\"It is, brahmin, because of attachment to views, adherence to views, fixation on views, addiction to views, obsession with views, holding firmly to views that ascetics fight with ascetics.\"

NDE\'s and skeptics

by David Turell @, Wednesday, September 02, 2009, 23:20 (5343 days ago) @ xeno6696

http://www.skepdic.com/nde.html 
 
> 
> "Reports of Christians meeting Muhammad or Muslims meeting Christ or Jews meeting Guru Nanak, if they exist, have not been publicized."
> 
> 
> [EDIT] And apparently Ketamine can be used to completely recreate the experience!-According to one researcher, and denied by another. THESE EXPERIENCES ARE VERY SIMILAR GENERALLY AS I NOTE BELOW. What is described by that website is questionable.-I've read many of the authors quoted in your skeptical website. Why does those skeptics twist what has been written by exclusion: In van Lommel's Lancet article, the man who lost his teeth when having a flat EKG and EEG is not mentioned by the skeptics. He recognized the face of the nurse who took them a week later when he was awake! He even described the room he was in. Remember his eyes were shut. -One of the most important parts of the article is missing from the skeptics description. Why? To shade the truth. The researchers prospectively graded the NDE's as from simple to complex and then the patients were followed for ensuing mortality. The complex ones had a duration time to mortality earlier, and statistically, significantly different than the simple ones.-I read many of the authors mentioned in the website: Moody, Sabom, Ring, Greyson, Blackmore, PMH Atwater, with whom I have had phone and email conversations. Blackmore is a typical half-baked skeptic who needs a lesson in human physiology. In dying to Live, 1993 half her objections were off the wall. I've had my patients talk to me about these. They are NOT hallucinations. I know one when I hear one, believe me. They are generally very similar. Many are 'good' but there are bad ones. People see only their own religion if religion is involved, which is not that common. They only see the dead or communicate with them. What your website does not allude to is the information some of these people receive during the experience that they could not have known beforehand, i.e. of another's recent death.-Moody challenged Sabom who did not believe Moody's assertions. Sabom (Cardiologist) started to inquire of his patients and became convinced of Moody's assertion. Do some real reading. Skeptical websites are untrustworthy when I compare what I know was written and reported, and what they describe as having been presented.

NDE\'s and skeptics

by xeno6696 @, Sonoran Desert, Wednesday, September 02, 2009, 23:47 (5343 days ago) @ David Turell

David,-Thanks for the reply and counter to the website. Note: I don't consider the experiences hallucinations, though I still strongly think that there is a physiological cause--the stuff dealing with Ketamine is a very strong counter argument. Other questions I had before the website, why do people only "experience" things based upon the religion they learned--why don't Christians see Muhammad? This suggests to me that these are personalized events that share more in common with dreams. -Case in point: I often experience what I call "deliberate deja-vu." Unfortunately it's always about something inane, but I'll dream about some event, such as part of a lecture I haven't had yet, remember the dream, and then suddenly it happens word for word down to the thoughts in my head. I still think there's a "worldy" explanation for that... just don't know where to begin. -Oh, as you (and dhw if he sees this) would be interested to know: I'm essentially accepted to grad school. My gpa of 3.42 wasn't looking at my transfer credits which bump me to 3.48 and on track to graduate with honors... as if I needed another reason to do well this semester. The graduate adviser told me submit my official app today which I did. She said it takes about 3 weeks for the paperwork to run its course.-I bought new GRE books for nothing, lol.-[EDIT]-As usual forgot something: If the guy was able to hear something while out, it could simply be the hearing that causes familiarity and the brain "thinks" it *saw* what it did. I bring up the memory and car crash scenario again...

--
\"Why is it, Master, that ascetics fight with ascetics?\"

\"It is, brahmin, because of attachment to views, adherence to views, fixation on views, addiction to views, obsession with views, holding firmly to views that ascetics fight with ascetics.\"

NDE\'s and skeptics

by David Turell @, Thursday, September 03, 2009, 17:41 (5342 days ago) @ xeno6696


> Thanks for the reply and counter to the website. -You always need to look at both sides. If you are skeptical, of course a skeptic website will support you, but you haven't learned anything. When my patients talked to me I was skeptical, until I read both sides.
 
> Case in point: I often experience what I call "deliberate deja-vu." I still think there's a "worldy" explanation for that... just don't know where to begin. -My wife does the same thing. It is a capacity in our brains. But how does it work? The man jumping from the 26th story was on television. Perhaps she picked up the signal. Electromagnetic waves could impinge on circuits in the brain. But what you are doing is advance viewing. And that raises the issue of human species consciousness. Work in this area is suggestive of its existence 
 
> The graduate adviser told me submit my official app today which I did. She said it takes about 3 weeks for the paperwork to run its course.-Fabulous!!!
> 
> 
> As usual forgot something: If the guy was able to hear something while out, -If his EEG is flat line he is not hearing. It takes a cortex for that. The famous case description is a lady with a very bad brain aneurysm. She was taken to deep hypothermia, but had purposeful noise with molded earpieces in both ears to keep track of brain function by EEG; 90 minutes after anethesia began and she was cooled down enough (60 degrees), eyes taped shut, she had an OBE in which she could describe an instrument used to cut the skull and she described a surgeon's conversaation about her small arteries. Skeptical?

NDE\'s and skeptics

by xeno6696 @, Sonoran Desert, Thursday, September 03, 2009, 19:46 (5342 days ago) @ David Turell


> > Thanks for the reply and counter to the website. 
> 
> You always need to look at both sides. If you are skeptical, of course a skeptic website will support you, but you haven't learned anything. When my patients talked to me I was skeptical, until I read both sides.
> -The hard part for me again, is the inability to negate perspective. I'd be willing to experience one myself so I can have a better grasp of what's going on. -> > Case in point: I often experience what I call "deliberate deja-vu." I still think there's a "worldy" explanation for that... just don't know where to begin. 
> 
> My wife does the same thing. It is a capacity in our brains. But how does it work? The man jumping from the 26th story was on television. Perhaps she picked up the signal. Electromagnetic waves could impinge on circuits in the brain. But what you are doing is advance viewing. And that raises the issue of human species consciousness. Work in this area is suggestive of its existence 
> -Forgive me if I say: that sounds more than a trifle Buddhist, lol. -> > The graduate adviser told me submit my official app today which I did. She said it takes about 3 weeks for the paperwork to run its course.
> 
> Fabulous!!!
> > 
> > 
> > As usual forgot something: If the guy was able to hear something while out, 
> 
> If his EEG is flat line he is not hearing. It takes a cortex for that. The famous case description is a lady with a very bad brain aneurysm. She was taken to deep hypothermia, but had purposeful noise with molded earpieces in both ears to keep track of brain function by EEG; 90 minutes after anethesia began and she was cooled down enough (60 degrees), eyes taped shut, she had an OBE in which she could describe an instrument used to cut the skull and she described a surgeon's conversaation about her small arteries. Skeptical?-I'm hard-programmed for skepticism, so of course. Nothing wrong with it, just means I'm a little more boring than most people, heh. -I think something was still sending signals. The brain may not have been working to process them (evidenced by the EEG) but that doesn't mean that cells are doing *nothing.* That, more than anything else lies at the center of my probing here. The only way those cells do *nothing* is when they're completely dead. -Is the mind the same thing as the soul? Question for a professional philosopher to be sure...

--
\"Why is it, Master, that ascetics fight with ascetics?\"

\"It is, brahmin, because of attachment to views, adherence to views, fixation on views, addiction to views, obsession with views, holding firmly to views that ascetics fight with ascetics.\"

NDE\'s and skeptics

by David Turell @, Thursday, September 03, 2009, 21:14 (5342 days ago) @ xeno6696

Is the mind the same thing as the soul? Question for a professional philosopher to be sure...-I have no idea. But in the Jewish religion both animals and people have souls. Transliterated: 'nefesh' for animal and 'nashama' for people. I could ask a Rabbi. :-)

NDE\'s and skeptics

by dhw, Friday, September 04, 2009, 08:45 (5341 days ago) @ xeno6696

David and Matt are discussing NDEs, and Matt has a theory that our brain operates "based on its own model at all instances. So when those pilots were walking 'behind themselves' this is because their brain's model isn't at a 1:1 correspondence to the world, they're still receiving sensory data but the brain is processing it incorrectly. In an OBE/NDE the theory was carried forward, suggesting that the brain was still receiving inputs and recording information that is then recalled upon awaking." (under Identity, 1 September at 22.55).-Matt also suggests that in an NDE "the low blood content of the brain in these instances breaks the barrier that allows our subconscious brain to take over the model, allowing you to have a 'waking dream' that could easily have you floating above yourself."-Thanks for the interesting website reference, and thanks also to David for his comprehensive analysis of it.-There are all kinds of weird experiences induced by drugs or excess alcohol, and there are illnesses that have the same effect. Whenever something disturbs the "1:1 correspondence", you're going to get weird results, and the experiment with the centrifuge doesn't seem any different to me. But some NDE experiences and some psychic experiences appear to result in the acquisition of knowledge to which the person concerned could not have had access. David has also mentioned this in his latest posts. These are the cases that really interest me, because if there are independent witnesses who can confirm the claims, we have a mystery on our hands. Two correspondents (David and BBella) on this website have had first-hand experience of such psychic phenomena, and my wife witnessed an instance that I've quoted in the "brief guide". (She knows of many more such instances back in her native Nigeria.) You can multiply these examples over and over again. The implication, if even just a small proportion of such cases is confirmed, is that there is an area of our identity or an area of reality that does not conform to any of the known patterns of the physical world. -The simplest "solution" to the mystery is to dismiss all such claims as being fraudulent, delusional, untestable etc. (I notice you still think there's a "worldly" explanation for your trivial déjà vu experiences.) That approach is TOO simplistic for me, because as I've said many times, I'm not prepared to dismiss people's personal experiences out of hand, above all when there is corroboration from third parties. This whole subject ties in with the difficult discussion I've been having with George on identity. David is convinced (as is BBella) that there is something beyond the physical world as we know it, and George is convinced that there isn't. I find it impossible to accept or reject either position because it seems to me that we just don't know enough to form a judgement. In particular, we don't know enough about Nature to draw a line between what might constitute "natural" and what might constitute "supernatural". So-called psychic powers may be lying dormant within all of us.-Meanwhile, congratulations on your good news! Your AgnosticWeb Supporters Club will raise a glass to you! Fingers crossed that the application will go through smoothly and you can save yourself a lot of time and hassle.

NDE\'s and skeptics

by xeno6696 @, Sonoran Desert, Friday, September 04, 2009, 13:13 (5341 days ago) @ dhw

David and Matt are discussing NDEs, and Matt has a theory that our brain operates "based on its own model at all instances. So when those pilots were walking 'behind themselves' this is because their brain's model isn't at a 1:1 correspondence to the world, they're still receiving sensory data but the brain is processing it incorrectly. In an OBE/NDE the theory was carried forward, suggesting that the brain was still receiving inputs and recording information that is then recalled upon awaking." (under Identity, 1 September at 22.55).
> 
> Matt also suggests that in an NDE "the low blood content of the brain in these instances breaks the barrier that allows our subconscious brain to take over the model, allowing you to have a 'waking dream' that could easily have you floating above yourself."
> 
> Thanks for the interesting website reference, and thanks also to David for his comprehensive analysis of it.
> 
> There are all kinds of weird experiences induced by drugs or excess alcohol, and there are illnesses that have the same effect. Whenever something disturbs the "1:1 correspondence", you're going to get weird results, and the experiment with the centrifuge doesn't seem any different to me. But some NDE experiences and some psychic experiences appear to result in the acquisition of knowledge to which the person concerned could not have had access. David has also mentioned this in his latest posts. These are the cases that really interest me, because if there are independent witnesses who can confirm the claims, we have a mystery on our hands. Two correspondents (David and BBella) on this website have had first-hand experience of such psychic phenomena, and my wife witnessed an instance that I've quoted in the "brief guide". (She knows of many more such instances back in her native Nigeria.) You can multiply these examples over and over again. The implication, if even just a small proportion of such cases is confirmed, is that there is an area of our identity or an area of reality that does not conform to any of the known patterns of the physical world. 
> -Amazing. Leave it to me to miss the obvious point and to you to tell me what it is. (Information from an "extra" sense.) -I suspect that there are some cases where this is true. However as you will point out shortly, we don't know enough...-> The simplest "solution" to the mystery is to dismiss all such claims as being fraudulent, delusional, untestable etc. (I notice you still think there's a "worldly" explanation for your trivial déjà vu experiences.) That approach is TOO simplistic for me, because as I've said many times, I'm not prepared to dismiss people's personal experiences out of hand, above all when there is corroboration from third parties. This whole subject ties in with the difficult discussion I've been having with George on identity. David is convinced (as is BBella) that there is something beyond the physical world as we know it, and George is convinced that there isn't. I find it impossible to accept or reject either position because it seems to me that we just don't know enough to form a judgement. In particular, we don't know enough about Nature to draw a line between what might constitute "natural" and what might constitute "supernatural". So-called psychic powers may be lying dormant within all of us.
> -Simplest if you're willing to dance on the experiences of others-which I'm not. It is a prime example of those situations that I complained about before in which -Yeah, I do think that there is an explanation for these things, and I think that it is a solvable mystery. I will add to your brief discussion about drugs and whatnot by reminding that nearly all native cultures use these experiences in a spiritual way. The altered state of the human mind in these instances are what leave me to believe that NDE's and OBE's have their roots in our brains. My dreams that predict inanities, why only when I'm asleep? I'd like to say its a more primitive brain at those hours, but knowing what I know about the brain leads me to say "don't be so sure of that." I know its in MY head and not someone else's, but that's about all I can say. -
> Meanwhile, congratulations on your good news! Your AgnosticWeb Supporters Club will raise a glass to you! Fingers crossed that the application will go through smoothly and you can save yourself a lot of time and hassle.-Hmm... as I think you mentioned living in the UK, would it be a Newcastle Brown? Either way, Cheers!

--
\"Why is it, Master, that ascetics fight with ascetics?\"

\"It is, brahmin, because of attachment to views, adherence to views, fixation on views, addiction to views, obsession with views, holding firmly to views that ascetics fight with ascetics.\"

NDE\'s and skeptics: new brain studies

by David Turell @, Friday, May 12, 2023, 23:47 (343 days ago) @ xeno6696

Following the brain in dying patients shows bursts of activity:

https://www.scientificamerican.com/article/surges-of-activity-in-the-dying-human-brain-...

"In their last minutes of life, some people's brains generate a surge of surprisingly organized-looking electrical activity that may reflect consciousness — although scientists aren't entirely sure.

"According to new research, published Monday (May 1) in the journal PNAS, this surge can sometimes occur after a person's breathing stops but before the brain stops functioning. The activity pattern is somewhat similar to what is seen when people are awake or in dreamlike states, leading to speculation that perhaps these electrical surges reflect the otherworldly experiences reported by people who've had close brushes with death: A sense of looking at the body from the outside; a tunnel and white light; or a sense of reliving important memories.

"However, since all the patients in the new study ultimately died, it's impossible to know if they had such experiences.

***

"Some people who are brought back from the brink of death report seeing or hearing unexplained things during resuscitation or when they seem to be unconscious. The reason for these near-death experiences is unknown, and it's not clear if they're even specific to death.

"International surveys suggest that only about half of what people call "near-death experiences" actually occur in life-threatening situations, said Daniel Kondziella, a neurologist at the University of Copenhagen who was not involved in the new research. The other half occur during meditation or in scary situations that don't endanger one's health or impact the brain's metabolism, Kondiziella told Live Science.

***

"Because the people who survive to report a near-death experience are inherently different from the people who die — their brains don't permanently lose function, for one thing — it's hard to determine whether those who actually die also have these subjective experiences.

"In 2013, Borjigin and her colleagues measured electrical activity in the brains of rats that they euthanized via cardiac arrest. They found that for about 30 seconds after the heart stopped, the brain showed a surge in what are called gamma waves, which are the highest-frequency electrical oscillations in the brain. Gamma waves are correlated with conscious experience, but don't necessarily prove that someone is conscious; they're just one of many indicators that someone might be aware and alert.

"In 2022, a separate group of doctors happened to be monitoring the brain of an 87-year-old man with an electroencephalogram (EEG), which detects electrical activity on the surface of the brain, when the man unexpectedly died. Similar to Borjigin's rats, the man's brain showed a surge in gamma activity in the 30 seconds before and after his heart stopped.

***

"The researchers got permission to monitor dying patients in intensive care whose breathing support had been removed after treatment proved futile. The study included four patients total, all of whom were comatose after cardiac arrest.

"In the 30 seconds to two minutes after their ventilators were removed, two of the four patients' brains showed surges in gamma waves. Interestingly, this gamma activity seemed organized, in that the gamma waves in one portion of the brain were associated with predictable activity patterns in other regions.

"The temporoparietal junction, a brain region where the temporal and parietal lobes meet, toward the back of the brain behind the ear, was particularly active with gamma waves. This region is known to be activated when people have out-of-body experiences or dreams, Borjigin said.

"The new findings echo what was seen in the 87-year-old patient who unexpectedly died, said Raul Vicente, a neuroscientist and data scientist at the University of Tartu who co-authored the 2022 study but was not involved in Borjigin's work. "It's very nice to see a confirmation," he told Live Science.

"'The more consistent findings we have, the more evidence it is that this likely is a mechanism happening at the time of death and if we can pinpoint this down to one location, even better," said Ajmal Zemmar, a neurosurgeon at the University of Louisville Health who also co-authored the 2022 study.

"Zemmar and Vicente are optimistic that these signals could be signs of conscious experience at the moment of death. But reflecting the debate in the field, Kondziella is more skeptical.

"'We know when you die a cardiac death as opposed to a brain death, that takes time," he said. Minutes pass between the heart stopping and brain cells dying, he said. "It shouldn't be a big surprise during those minutes, you will see aberrant electrophysiological activity in the brain.'"

Comment: great studies but no answers. We are faced with living testimony of vast near-to-death experiences, with no good physical evidence. No one has had an EEG running during a true NDE episode. Would be nice to have one. If only the 87-year-old lived.

NDE\'s and skeptics: Ethan Alexander's new interview

by David Turell @, Thursday, July 27, 2023, 20:56 (267 days ago) @ David Turell

Effects of DMT discussed:

https://www.realclearscience.com/blog/2023/07/26/neurosurgeon_compares_his_near-death_e...

"More recently, Alexander’s popular account of his near-death experience attracted the attention of Pascal Michael, a PhD student in psychology at the University of Greenwich. Michael met Alexander after seeing him speak at an academic conference and was informed that Alexander had experimented with 5-MeO-DMT, a psychedelic closely related to DMT, secreted from the glands of the Colorado River toad.

***

"In the interview, Michael asked Alexander to deeply describe both experiences. Per Alexander’s recollections, both evoked feelings of transcending time and space and glimpsing the multiverse, while at the same time eliciting a sense of unity with his surroundings, as well as a profound — almost divine — love for everything and everyone.

"The most vital similarity between Alexander’s near-death experience and his 5-MeO-DMT trip was “ego death,” the dissolution of his sense of self. “The annihilation, though temporary, of the sense of one’s individuated self and all of its concomitant autobiographical memories often gives rise to an experience of being a ‘cosmic being,'” Michael explained. This, he noted, is key to the psychedelic-induced mystical experience and any potential benefits extracted from it.

"Alexander also described many differences between the two events, avowing that during his near-death experience he came across both menacing and divine beings, and even briefly ventured to another world. At one point, he also passed through an “abyssal emptiness and yet suffused with nurturing light” and came upon a “threshold” which a feminine entity prevented him from crossing.

"Due to these dissimilarities, and the fact that, at least for him, the near-death experience vastly surpassed that of the psychedelic, Alexander does not ascribe to the theory that DMT causes near-death experiences, preferring a transcendental explanation – that he briefly glimpsed some form of afterlife.

“'[The 5MeO was] like looking through a little peephole, as opposed to being full-bore swimming and being immersed in the Pacific ocean of being completely into that oneness experience [of the NDE],” he told Michael.

"Michael notes, however, that all the contrasting moments in Alexander’s near-death experience are broadly characteristic of DMT psychedelic trips. So Alexander’s episode by no means disproves the broader theory that near-death experiences could be caused by the body’s internally produced psychedelic compounds. Nor does it confirm it."

Comment: having read Alexander's book I can understand why he discredits the psychedelic drug. Especially with later conformation of what Alesander learned about himself.

NDE\'s and skeptics: Parnia's latest studies

by David Turell @, Saturday, September 16, 2023, 19:45 (216 days ago) @ David Turell

From a large group of patients:

https://www.scientificamerican.com/article/some-patients-who-died-but-survived-report-l...

"According to findings published on September 14 in Resuscitation, the flatlined brains of some cardiac arrest patients burst into a flurry of activity during CPR, even though their heart stopped beating up to an hour. A small subset of study participants who survived were able to recall the experience, and one person was able to identify an audio stimulus that was played while doctors were trying to resuscitate them.

"The researchers interpret the brain recordings they made of these patients as markers of “lucid, recalled experiences of death”—an observation that has “never been possible before,” says lead author Sam Parnia, an associate professor of medicine at NYU Langone Health and a longtime researcher of what happens to people as they die. “We’ve also been able to put forward a coherent, mechanistic explanation for why this occurs.”

***

"Between May 2017 and March 2020, 567 people suffered cardiac arrests at participating hospitals. Medical staff managed to gather usable brain oxygen and activity data from 53 of these patients, most of whom showed an electrical flatline state on electroencephalographic (EEG) brain monitors. But about 40 percent then experienced electrical activity that reemerged at some point with normal to near-normal brain waves that were consistent with consciousness. This activity was sometimes restored up to 60 minutes into CPR.

"Of the 567 total patients, just 53 survived. The researchers conducted interviews with 28 of the survivors. They also interviewed 126 people from the community who had gone through cardiac arrests because the sample size of survivors from the new study was so small. Nearly 40 percent reported some perceived awareness of the event without specific memories attached, and 20 percent seemed to have had a recalled experience of death. Many in the latter group described the event as a “moral evaluation” of “their entire life and how they’ve conducted themselves,” Parnia says.

***

"He and his colleagues have developed a working hypothesis to explain their findings. Normally, the brain has “braking systems” in place that filter most elements of brain function out of our experience of consciousness. This enables people to efficiently operate in the world, because under regular circumstances, “you couldn’t function with access to your whole brain’s activity being in the realm of consciousness,” he says.

"In the dying brain, however, the researchers hypothesize that the braking system is removed. Parts that are normally dormant become active, and the dying person gains access to their entire consciousness—“all your thoughts, all your memories, everything that’s been stored before,” Parnia says. “We don’t know the evolutionary benefit of this, but it seems to prepare people for their transition from life into death.”

"The findings also raise questions about the brain’s resiliency to oxygen deprivation. It could be, Parnia says, that some people who have conventionally been thought to be beyond the point of saving could in fact be revived. “The traditional thinking among doctors is that the brain, once deprived of oxygen for five to 10 minutes, dies,” he says. “We were able to show that the brain is quite robust in terms of its ability to resist oxygen deprivation for prolonged periods of time, which opens up new pathways for finding treatments for brain damage in the future.”

"The new study “represents a Herculean effort to understand as objectively as possible the nature of brain function as it may apply to consciousness and near-death experiences during cardiac arrest,” says Lakhmir Chawla, an intensive care unit physician at Jennifer Moreno Department of Veterans Affairs Medical Center in San Diego, Calif., who was not involved in the research but has published papers on spikes of EEG activity at the time of death in some patients."

Comment: this study assumes that what the patient experiences in directly related to brain activity. This is in sharp contrast to Eben Alexander, a neurosurgeon who experienced prolonged coma with no EEG activity yet had a prolonged experience in what he recalled as 'heaven' with a guardian angel who looked exactly like his sister. Since he was adopted, he had never known her in his life. Research after his recovery revealed her.

NDE\'s and skeptics: Parnia's latest studies

by David Turell @, Thursday, December 14, 2023, 16:24 (127 days ago) @ David Turell

Another article on same study:

https://www.popularmechanics.com/science/health/a45632815/brain-activity-after-death/?s...

"Using an experiment design they began developing in 2015—five years before the final, full version of the study was finally completed—the scientists rigged up people who experienced cardiac arrest. This carefully performed study is part of an area of medicine and research focused on resuscitation, using methods like CPR and defibrillation. And due to the sensitive nature of trying to study people who are experiencing cardiac arrest, something that only about 10 percent of patients survive, this kind of work has a ton of value for people who are interested in near-death experiences and other edge cases of brain activity.

***

"In the study, just 28 people of 567 total both survived cardiac arrest and were able to complete the cognitive evaluations and surveys. Altogether, 4 in 10 of those patients recalled “some degree of consciousness” following CPR. Of those 28, 11 people said they had memories or similar sensations from during their minutes of cardiac arrest. Six people, or 21.4 percent of those surveyed, experienced “transcendent recalled experience of death (RED).” Three reported something like dreams. None of the people exhibited signs that they were actually conscious, like moving around.

"One patient said they’d seen their father. One felt they were standing next to their bed and watching their body receive CPR. One heard their deceased grandmother tell them to “go back.” Some remembered the moments before they were finally revived into consciousness. The dreams were, well, dreams: “Then, I walked into a puddle… When I got out of the puddle, I was not wet and I sort of melded into the pavement…”

***

"The attempts to guide or influence memories using images or sounds were almost completely unsuccessful. One person out of 28 identified the three spoken words, and none identified the images. In a way, this reinforces that what was happening was deep inside the brain. It points to a way that some kind of consciousness may endure even when, so to speak, the lights are out in the rest of the brain."

Comment: this artificial attempt to intrude into NDE's did not work but the studies did demonstrate deep brain activity while unconscious, indicating a way the brain could receive or develop NDE's.

Identity

by dhw, Sunday, August 30, 2009, 08:21 (5346 days ago) @ xeno6696

Matt: When Fred takes his demon story to me, all I can do is provide what I think based on my perspective. However, it is two things, firstly it is my examination based on his words, which may or may not be wholly accurate. Secondly, his perspective shapes his retelling of the events, and further removes me from the actual event itself. There is far too much subjectivity here to be able to study it. - An excellent analysis, if I may say so, and I would react to Fred's demons in precisely the same way as you. We needn't argue about the implications of "study", as you've defined what you mean and my own definition would simply be a little less rigid. In fact, there is not really any dispute here at all. I had pointed out that one difference between us was that I was not prepared to dismiss personal experiences as possible evidence, and you have explained the reasons for your scepticism. I am also sceptical, both of materialism and of the "supernatural", but since there are so many unexplained mysteries, there is a kind of "maybe and maybe not" behind my thinking. Perhaps, though, if I underline possible in "possible evidence", it will reduce the gap between us!

Identity

by xeno6696 @, Sonoran Desert, Sunday, August 30, 2009, 18:29 (5346 days ago) @ dhw

dhw, - > Matt: When Fred takes his demon story to me, all I can do is provide what I think based on my perspective. However, it is two things, firstly it is my examination based on his words, which may or may not be wholly accurate. Secondly, his perspective shapes his retelling of the events, and further removes me from the actual event itself. There is far too much subjectivity here to be able to study it.
> 
> An excellent analysis, if I may say so, and I would react to Fred's demons in precisely the same way as you. We needn't argue about the implications of "study", as you've defined what you mean and my own definition would simply be a little less rigid. In fact, there is not really any dispute here at all. I had pointed out that one difference between us was that I was not prepared to dismiss personal experiences as possible evidence, and you have explained the reasons for your scepticism. I am also sceptical, both of materialism and of the "supernatural", but since there are so many unexplained mysteries, there is a kind of "maybe and maybe not" behind my thinking. Perhaps, though, if I underline possible in "possible evidence", it will reduce the gap between us! - I've been saying for some time that we have more in common than not. Unfortunately the common ground is that sometimes infuriating "I don't know." I bet that our differences are probably quite small... there's a few more things I accept on the materialist side but based mainly upon methodology. I probably seem more pessimistic because of that.

--
\"Why is it, Master, that ascetics fight with ascetics?\"

\"It is, brahmin, because of attachment to views, adherence to views, fixation on views, addiction to views, obsession with views, holding firmly to views that ascetics fight with ascetics.\"

Identity

by George Jelliss ⌂ @, Crewe, Wednesday, August 26, 2009, 20:29 (5350 days ago) @ dhw

Concerning the issue of "free will" I've recently come across these ideas that may be relevant: - Perceptual control theory - http://www.brainstorm-media.com/users/powers_w/whatpct.html - Quote from near the end: - "The explanation is simply that the planning level of the brain plans not actions, but perceptions." /// "These are requests for perceptions, not commands for actions. They are, in fact, reference signals. The higher system, once it has specified what the control system is to perceive, has no more to do with creating those perceptions. The closed causal loop takes care of operating the steering wheel" [i.e. controls] "so that the actual perceptions continue to match the reference perceptions, even as the reference perceptions change. At the same time, it alters the actions as needed to counteract the effects of any and all disturbances that could make the perception depart from the reference perception." - Also: - http://www.perceptualcontroltheory.org/overview.html - Quote:
"PCT stands in contrast to the stimulus-response model of behavior associated with B. F. Skinner, and to the cognitive science model. These older models attempt to explain behavior either do not recognize or do not clearly understand control. Behavior is neither just caused by stimuli in the environment nor is it blind execution of internal plans. Behavior is not an end result. It is an integral part of the closed loop process which controls perception."

--
GPJ

Identity

by xeno6696 @, Sonoran Desert, Friday, August 28, 2009, 03:11 (5349 days ago) @ George Jelliss

Concerning the issue of "free will" I've recently come across these ideas that may be relevant:
> 
> Perceptual control theory
> 
> http://www.brainstorm-media.com/users/powers_w/whatpct.html
> 
> Quote from near the end:
> 
> "The explanation is simply that the planning level of the brain plans not actions, but perceptions." /// "These are requests for perceptions, not commands for actions. They are, in fact, reference signals. The higher system, once it has specified what the control system is to perceive, has no more to do with creating those perceptions. The closed causal loop takes care of operating the steering wheel" [i.e. controls] "so that the actual perceptions continue to match the reference perceptions, even as the reference perceptions change. At the same time, it alters the actions as needed to counteract the effects of any and all disturbances that could make the perception depart from the reference perception." 
> 
> Also:
> 
> http://www.perceptualcontroltheory.org/overview.html
> 
> Quote:
> "PCT stands in contrast to the stimulus-response model of behavior associated with B. F. Skinner, and to the cognitive science model. These older models attempt to explain behavior either do not recognize or do not clearly understand control. Behavior is neither just caused by stimuli in the environment nor is it blind execution of internal plans. Behavior is not an end result. It is an integral part of the closed loop process which controls perception." - George, - Thanks for these links, it's nice to see that someone has at least worked to resolve some of my issues in this area. This is a promising model.

--
\"Why is it, Master, that ascetics fight with ascetics?\"

\"It is, brahmin, because of attachment to views, adherence to views, fixation on views, addiction to views, obsession with views, holding firmly to views that ascetics fight with ascetics.\"

Identity

by dhw, Friday, August 28, 2009, 08:39 (5348 days ago) @ George Jelliss

My thanks to George for pointing out two articles on Perceptual Control Theory.
Unfortunately, the first one practically drove me round the bend in more senses than one, and it was extremely kind of you to give us the summarizing quote from near the end. Perhaps I should have taken the hint, and not tried to navigate my way through the rest! The second article is a nicely succinct summary. However, PCT appears only to deal with behaviour ... for instance, how we process stimuli and fit them in with preplanned intentions. Interesting in itself, but no help with the problems I'm trying to get to grips with under this thread. I appreciate the references, though, George.

Identity

by xeno6696 @, Sonoran Desert, Saturday, September 05, 2009, 18:14 (5340 days ago) @ dhw

Somewhat related to the subject of Identity:-http://www.sciencedaily.com/releases/2009/09/090904071908.htm-Someone is attempting to take on the task of unifying all of the known neuroscientific data in a project to... make a human brain.

--
\"Why is it, Master, that ascetics fight with ascetics?\"

\"It is, brahmin, because of attachment to views, adherence to views, fixation on views, addiction to views, obsession with views, holding firmly to views that ascetics fight with ascetics.\"

Identity

by dhw, Sunday, February 07, 2010, 14:59 (5185 days ago) @ dhw

An article in The Guardian last week described how doctors in Belgium have communicated with a man believed to have been in a vegetative state for seven years, since suffering a traumatic brain injury in a car accident. Using a hi-tech scanner, they were able to ask him questions and monitor his brain activity. In order to answer yes, he had to imagine playing tennis, and to answer no he was told to think of wandering through his house. "We were astonished when we saw the results of the patient's scan and that he was able to correctly answer the questions that were asked by simply changing his thoughts," said Dr Adrian Owen, assistant director of the Medical Research Council's cognition and brain sciences unit at Cambridge University.-One's imagination can hardly take in the horror of lying in a bed for seven years, aware of what's going on, trapped inside your own head, unable to show people that you are still there. I think I'd rather be dead.-However, such discoveries once more highlight the question of what constitutes consciousness and identity. Here is a man whose brain is so badly damaged that he cannot move or actively communicate anything. And yet he has memory, he can respond to questions, and at will he can summon up specific images which spark off chemical reactions that can be monitored. It may be argued that the relevant sections of the brain must have remained undamaged, but what is this "will"? And what constitutes the identity of the person that directs the will that directs the mind to form such images, if the will, memory and mind exist only as material cells?

Identity

by David Turell @, Sunday, February 07, 2010, 15:07 (5185 days ago) @ dhw

An article in The Guardian last week described how doctors in Belgium have communicated with a man believed to have been in a vegetative state for seven years, since suffering a traumatic brain injury in a car accident. Using a hi-tech scanner, they were able to ask him questions and monitor his brain activity. In order to answer yes, he had to imagine playing tennis, and to answer no he was told to think of wandering through his house. "We were astonished when we saw the results of the patient's scan and that he was able to correctly answer the questions that were asked by simply changing his thoughts," said Dr Adrian Owen, assistant director of the Medical Research Council's cognition and brain sciences unit at Cambridge University.
> 
> One's imagination can hardly take in the horror of lying in a bed for seven years, aware of what's going on, trapped inside your own head, unable to show people that you are still there. I think I'd rather be dead.
> 
> However, such discoveries once more highlight the question of what constitutes consciousness and identity. Here is a man whose brain is so badly damaged that he cannot move or actively communicate anything. And yet he has memory, he can respond to questions, and at will he can summon up specific images which spark off chemical reactions that can be monitored. It may be argued that the relevant sections of the brain must have remained undamaged, but what is this "will"? And what constitutes the identity of the person that directs the will that directs the mind to form such images, if the will, memory and mind exist only as material cells?

Identity

by David Turell @, Sunday, February 07, 2010, 15:15 (5185 days ago) @ David Turell

An article in The Guardian last week described how doctors in Belgium have communicated with a man believed to have been in a vegetative state for seven years, since suffering a traumatic brain injury in a car accident. Using a hi-tech scanner, they were able to ask him questions and monitor his brain activity. 
> > However, such discoveries once more highlight the question of what constitutes consciousness and identity. Here is a man whose brain is so badly damaged that he cannot move or actively communicate anything. And yet he has memory, he can respond to questions, and at will he can summon up specific images which spark off chemical reactions that can be monitored. It may be argued that the relevant sections of the brain must have remained undamaged, but what is this "will"? And what constitutes the identity of the person that directs the will that directs the mind to form such images, if the will, memory and mind exist only as material cells?-I cannot answer your questions, but before scanners, eye movements identified patients in a so-called 'locked-in' state. Blink once for 'yes' and twice for 'no' sort of games. It appears to me the whole personage of his consciousness is inside, locked-in, but functional. Moving physically is in the 'motor strip' on both sides. That is not where personality hides or thoughts are processed or memory recovered.

RSS Feed of thread
powered by my little forum